Site Loader

Содержание

Скорость света 🚀 — что это? Чему равна в вакууме / воздухе?

Точные значения скорости света

метров в секунду

299 792 458

Приблизительные значения скорости света

километров в секунду

300 000

километров в час

1,08 млрд

миль в секунду

186 000

миль в час

671 млн

Скорость света: чему она равна и как ее измерять

Скорость света — это величина, характеризующая быстроту перемещения света.

До второй половины XVII века скорость света считалась бесконечной, пока ее не измерил датский астроном Олаф Рёмер. Он наблюдал затмения спутника Юпитера Ио и заметил, что они не совпадают по времени с расчетными, а зависит это несовпадение от расстояния между событием и наблюдателем. Принимая во внимание положение Земли на своей орбите относительно Юпитера, Рёмер подсчитал, что скорость света равна 220 000 км/с.

В начале XIX века французский ученый Физо разработал для измерения скорости света так называемый метод прерываний. Физик направил луч света на зеркало. Отражаясь от него, свет проходил через зубцы колеса. Затем попадал на еще одну отражающую поверхность, которая была расположена на расстоянии в 8,6 км. Колесо вращали, увеличивая скорость, пока луч не будет видно в следующем зазоре. После подсчетов Физо получил результат — 313 000 км/с.


Изобретение лазера в XX веке позволило дойти до предела точности и зафиксировать скорость света на отметке 299 792 458 м/с с погрешностью 1,2 м/c. Дальнейшее уточнение стало невозможным из-за отсутствия точного определения метра. В то время за эталон брали металлическую палку, хранящуюся в палате мер и весов.

В восьмидесятых годах прошлого века Генеральная конференция по мерам и весам (да, такая действительно существует) приняла за метр расстояние, которое преодолевает свет за 1/299 792 458 секунды. Соответственно, скорость света стала официально равной 299 792 458 метров в секунду. Для удобства ее значение принято округлять до 300 000 км/с.

Курсы подготовки к ОГЭ по физике помогут снять стресс перед экзаменом и получить высокий балл.

Неудавшийся опыт Галилея

Чтобы измерить скорость света, в 1600 году Галилей и его помощник взобрались на соседние холмы, предварительно рассчитав расстояние между ними. Они взяли зажженные фонари и оборудовали их заслонками, которые открывают и закрывают огни. Поочередно открывая и закрывая огонь, они пытались рассчитать скорость света. Галилей и помощник заранее знали, с какой задержкой будут открывать и закрывать огонь. Когда один из них открывал заслонку, то же должен был сделать и другой.

Однако эксперимент был провальным, и неудивительно: чтобы все получилось, ученым пришлось бы стоять на расстоянии в миллионы километров друг от друга.

Практикующий детский психолог Екатерина Мурашова

Бесплатный курс для современных мам и пап от Екатерины Мурашовой. Запишитесь и участвуйте в розыгрыше 8 уроков

Скорость света в различных средах

Свет распространяется в разных средах по-разному. В вакууме и в воздухе скорость света почти не различается, а вот в других средах она меньше. Это зависит от оптической плотности среды — чем она больше, тем меньше скорость распространения света.

Основной характеристикой в данном случае служит показатель преломления среды. Он равен отношению скорости света в вакууме к скорости распространения света в среде.

Абсолютный оказатель преломления среды

n = c/v

n — показатель преломления среды [—]

с — скорость света [м/с]

v — скорость света в заданной среде [м/с]

Ниже представлена таблица скоростей света в разных средах и показателей преломления в них.

Среда

Скорость света, км/с

Абсолютный показатель преломления среды

Вакуум

300 000

1

Воздух

299 704

1,003

Лед

228 782

1,31

Вода

225 341

1,33

Стекло

200 000

1,5

Сахар

192 300

1,56

Сероуглерод

184 000

1,63

Рубин

170 386

1,76

Алмаз

123 845

2,42

Параметры, связанные со скоростью света

Самые важные параметры — это длина волны и период.

Формула скорости света

c = λ/T

с — скорость света [м/с]

λ — длина волны [м]

T — период [с]

Задачка для практики

Определите цвет освещения, проходящий расстояние в 1000 раз больше его длины волны за 2 пикосекунды.

Решение

Для начала переведем 2 пикосекунды в секунды — это 2 * 10-12 с.

Теперь возьмем формулу скорости: v = S/t

По условию S = 1000λ, то есть v = 1000λ/t.

Выражаем длину волны:

λ = vt/1000

Подставляем значения скорости света и известного нам времени:

λ = (3 * 108 * 2 * 10-12)/1000 = 600

И соотносим со шкалой видимого света:


На шкале видно, что длине волны в 600 нм соответствует оранжевый цвет излучения.

Ответ: цвет освещения при заданных условиях будет оранжевым.

Учёба без слёз (бесплатный гайд для родителей)

Пошаговый гайд от Екатерины Мурашовой о том, как перестать делать уроки за ребёнка и выстроить здоровые отношения с учёбой.

Скорость выше, чем скорость света

Здесь мы подходим к самому интересному. По сути, преодолеть скорость света — это то же самое, что изобрести машину времени. Ведь мы не можем увидеть свет от зажженного на улице фонаря раньше, чем он зажегся. Казалось бы, вопрос закрыт, машина времени невозможна и вообще все мечты детства разрушены. Но на самом деле это не совсем так.

Физически машину времени ничто не запрещает. То есть с точки зрения физики она вполне возможна, у нас есть только технические ограничения.

Согласно общей теории относительности, чем быстрее мы разгоняем частицу, у которой есть некая масса, тем больше энергии нам требуется. По мере приближения к скорости света эта энергия будет стремиться к бесконечности. Но это не означает, что свет на порядки быстрее всего во Вселенной. Например, ученые ЦЕРНа разогнали протоны в Большом адронном коллайдере до скорости 299 792 455 м/c, что всего на 3 м/с уступает невесомым фотонам света.

Описанные выше ограничения, которые накладывает на скорости во Вселенной современная физика, не касаются частиц, которые не имеют массы, не взаимодействуют с обычными частицами и могут перемещаться быстрее скорости света. Такие частицы принято называть тахионами и на данный момент их существование является лишь предположением (сложно придумать эффективный инструмент для их обнаружения, ведь они ни с чем не взаимодействуют).

В специальной теории относительности есть даже такое понятие, как релятивистское замедление времени. Его смысл заключается в том, что в движущемся теле все физические процессы проходят медленнее.

Классическим примером этого явления является сценарий близнецов. Представим, что один близнец летит на космическом корабле со скоростью, близкой к скорости света, а другой остается на Земле. Когда близнец-космонавт вернется на Землю постаревшим всего на год или на два, он обнаружит, что его брат стал старше на несколько десятилетий.

В реальной жизни эксперимент с близнецами никто не проводил, но проводили аналогичный — с часами. Ученые запустили атомные часы на орбиту и оставили идентичные часы на Земле. Когда часы вернулись, они шли с некоторым отставанием от своего земного близнеца.

Еще один популярный пример сверхсветовой скорости — это явления квантовой механики. В тот самый момент, когда вы надели на правую ногу один носок, второй моментально и автоматически стал левым, несмотря на расстояние между ними.

Или эксперимент с котом Шрёдингера, про который вы наверняка что-то слышали.

Лирическое отступление про кота Шрёдингера

Физик, которому не очень нравятся кошки, помещает кота в коробку вместе с бомбой, которая взрывается с вероятностью 50% после того, как закрыли крышку. До того, как мы откроем коробку, нет способа узнать, взорвалась ли бомба.

Поэтому мы не знаем, жив кот или мертв.

Оперируя понятиями квантовой физики, мы можем сказать, что до нашего наблюдения кот находился в состоянии суперпозиции — состоянии, сочетающем в себе обе возможности с шансом 50% для каждой.

Нечто подобное случается с физическими системами квантовых размеров, вроде электрона, вращающегося вокруг атома водорода. Электрон не совсем вращается — он как бы находится во всем пространстве одновременно, а в некоторых местах с большей вероятностью. Только после того, как мы определили его местоположение, мы можем точно указать, где он находится в этот момент. Так же, как мы не знали, был кот жив или мертв до того, как мы открыли коробку.

Это подводит нас к странному и красивому феномену квантовой запутанности. Представим себе, что вместо одного кота в одной коробке у нас было бы два кота в двух разных коробках. Если мы повторим эксперимент с котом Шрёдингера с парой этих котов, в результате эксперимента могут быть четыре возможности:

  • оба кота будут живы,
  • оба мертвы,
  • первый будет жив, второй мертв,
  • первый мертв, второй жив.

Ситуации, когда оба кота мертвы или оба кота живы, не соответствуют состоянию суперпозиции. Другими словами, возможна такая система из двух котов, в которой в итоге всегда один из котов будет мертв, а другой жив. Пользуясь техническими терминами, можно сказать, что состояния этих двух котов запутаны.

Назревает вопрос: что произойдет, если этих котов поместить в разных уголках Вселенной. Не поверите, но то же самое! Один из котов в любом случае будет жив, а другой — мертв, хотя какой конкретно кот будет жив, а какой мертв, совершенно непредсказуемо.

Квантовая запутанность была подтверждена в настоящих лабораторных экспериментах. Две субатомные частицы запутаны в состоянии суперпозиции так, что если одна вращается в одну сторону, то другая — в противоположную.

Запутанность находится в центре квантовой информатики — развивающейся области науки, которая ищет применение законам странного квантового мира. Так, квантовая криптография позволяет шпионам надежно посылать друг другу информацию, а квантовое программирование — взламывать секретные коды.

Каждодневная физика со временем может стать более похожей на странный мир квантовой механики. Квантовая телепортация сможет достигнуть такого прогресса, что однажды ваш кот сможет сбежать в более безопасную вселенную, где нет физиков и коробок.

В общем, сверхсветовая скорость существует, хоть у нее и очень слабая доказательная база. Если ученые добьются того, чтобы скорости выше скорости света стали нашей реальностью, то и до машины времени недалеко.

Скорость света в вакууме равна 299 792 458 метров в секунду (м/с)

 ГЛАВНАЯ  »  МАТЕРИАЛЫ  »  Какая скорость света в вакууме?

5 февраля, 2017 МАТЕРИАЛЫ 2 комментария

Скорость света в различных средах различается значительно. Сложность состоит в том, что человеческий глаз не видит его во всем спектральном диапазоне. Природа происхождения световых лучей интересовала ученых еще в древности. Первые попытки расчета скорости света были предприняты еще за 300 лет до н.э. В тот период ученые определили, что волна распространяется по прямой линии.

Быстрый ответ

Им удалось описать математическими формулами свойства и света и траекторию его движения. Скорость света в вакууме стала известной через 2 тысячи лет после проведения первых исследований.

Световой луч представляет собой электромагнитную волну в сочетании с фотонами. Под фотонами понимают простейшие элементы, которые также называют квантами электромагнитного излучения. Световой поток во всех спектрах невидим. Он не перемещается в пространстве в традиционном понимании этого слова. Для описания состояния электромагнитной волны с квантовыми частицами введено понятие показателя преломления оптической среды.

Световой поток переносится в пространстве в виде луча с малым поперечным сечением. Способ движения в пространстве выведен геометрическими методами. Это прямолинейный пучок, который на границе с различными средами начинает преломляться, формируя криволинейную траекторию. Ученые доказали, что максимальная скорость создается в вакууме, в других средах скорость движения может различаться в разы. Учеными разработана система, световой луч и выведенная величина в которой является основной для выведения и отсчета некоторых единиц СИ.

Немного исторических фактов

Примерно около 900 лет назад Авиценой было выдвинуто предположение, что независимо от номинала величины скорость света имеет конечное значение. Галилео Галилей пытался опытным путем вычислить скорость светового потока. С помощью двух фонариков экспериментаторы пытались засечь время, за которое световой пучок от одного объекта будет виден другому. Но такой эксперимент выявился неудачным. Скорость оказалась столь высока, что им не удалось засечь время задержки.

Галилео Галилей обратил внимание на то, что у Юпитера промежуток между затмениями четырех его спутников составил 1320 секунд. На основе этих открытий в 1676 году астроном из Дании Оле Ремер рассчитал скорость распространения светового пучка, как значение 222 тысячи км/сек. На тот период данное измерение было наиболее точным, но его не могли проверить земными мерками.

Через 200 лет Луизи Физо смог вычислить скорость движения светового луча опытным путем. Он создал специальную установку с зеркалом и зубчатым механизмом, который вращался на огромной скорости. Световой поток отражался от зеркала и через 8 км возвращался назад. При увеличении скорости колеса возникал тот момент, когда зубчатый механизм перекрывал луч. Таким образом, скорость луча была установлена, как 312 тысяч километров в секунду.

Фуко усовершенствовал это оборудование, уменьшив параметры за счет замены зубчатого механизма плоским зеркалом. У него точность измерений получилась наиболее приближенной к современному эталону и составила 288 тысяч метров в секунду. Фуко предпринял попытки рассчитать скорость света в инородной среде, взяв за основу воду. Физику удалось сделать вывод, что данная величина не постоянная и зависит от особенностей преломления в данной среде.

Чему равна скорость света в вакууме?

Вакуум представляет собой пространство, свободное от вещества. Скорость света в вакууме в системе Си обозначена латинской буквой C. Она является недостижимой. Ни один предмет нельзя разогнать до такого значения. Физики только предполагают, что может произойти с объектами, если они разгонятся до такой степени. Скорость распространения светового луча обладает постоянными характеристиками, она:

  • постоянная и конечная;
  • недостижимая и неизменная.

Знание этой константы позволяет вычислить, с какой максимальной скоростью объекты могут перемещаться в космосе. Величина распространения луча света признана фундаментальной постоянной. Она используется для характеристик пространства времени. Это предельно допустимое значение для движущихся частиц. Какая скорость света в вакууме? Современную величину получили посредством лабораторных измерений и математических подсчетов. Она равна 299.792.458 метров в секунду с точностью до ± 1,2 м/с. Во многих дисциплинах, в том числе в школьных, при решении задач используются приближенных вычисления. Берется показатель, равный 3•108 м/с.

Световые волны видимого человеку спектра и рентгеновские волны возможно разогнать до показаний, приближающихся до скорости распространения света. Они не могут сравняться с этой константой, а также превысить ее значение. Константа выведена на основе отслеживания поведения космических лучей в момент разгона их в специальных ускорителях. Она зависит от той инерциальной среды, в которой происходит распространение луча. В воде прохождение света ниже на 25%, а воздухе будет зависеть от температуры и давления на момент вычислений.

Все расчеты проведены с использованием теории относительности и закону причинности, выведенному Энштейном. Физик считает, что если объекты достигнут скорости 1 079 252 848,8 километров/час и превысят ее, то произойдут необратимые изменения в строении нашего мира, система поломается. Время начнет отсчитываться в обратном порядке, нарушая порядок событий.

На основе скорости светового луча выведено определение метра. Под ним понимают участок, который успевает пройти световой луч за 1/299792458 секунды. Не следует смешивать данное понятие с эталоном. Эталон метра — это специальное техническое устройство на кадмиевой основе со штриховкой, позволяющее видеть данное расстояние физически.

Скорость света в вакууме

Скорость света в вакууме в представлении художника

Содержание:

  • 1 История
    • 1.1 Уравнения Джеймса Максвелла – электромагнитная природа света
    • 1.2 Опыт Майкелъсона — абсолютность скорости света
    • 1.3 Специальная теория относительности Альберта Эйнштейна
  • 2 Материалы по теме
    • 2.1 Парадокс относительности одновременности СТО
  • 3 Как измеряют скорость света?
    • 3.1 Наблюдения Олаф Рёмера
    • 3.2 Наблюдения Джеймса Брэдли
    • 3.3 Опыт Луи Физо
    • 3.4 Дальнейшие измерения
  • 4 Подведем итоги

История

В XIX веке произошло несколько научных экспериментов, которые привели к открытию ряда новых явлений. Среди этих явлений – открытие Гансом Эрстедом порождения магнитной индукции электрическим током. Позже Майкл Фарадей обнаружил обратный эффект, который был назван электромагнитной индукцией.

Уравнения Джеймса Максвелла – электромагнитная природа света

В результате этих открытий было отмечено так называемое «взаимодействие на расстоянии», в результате чего новая теория электромагнетизма, сформулированная Вильгельмом Вебером, была основана на дальнодействии. Позже, Максвелл определил понятие электрического и магнитного полей, которые способны порождать друг друга, что и есть электромагнитной волной. Впоследствии Максвелл использовал в своих уравнениях так называемую «электромагнитную постоянную» — с.

К тому времени ученые уже вплотную приблизились к тому факту, что свет имеет электромагнитную природу. Физический же смысл электромагнитной постоянной – скорость распространения электромагнитных возбуждений. На удивление самого Джеймса Максвелла, измеренное значение данной постоянной в экспериментах с единичными зарядами и токами оказалось равным скорости света в вакууме.

До данного открытия человечество разделяло свет, электричество и магнетизм. Обобщение Максвелла позволило по-новому взглянуть на природу света, как на некий фрагмент электрического и магнитного полей, распространяющийся самостоятельно в пространстве.

На рисунке ниже изображена схема распространения электромагнитной волны, которой также является свет. Здесь H – вектор напряженности магнитного поля, E — вектор напряженности электрического поля. Оба вектора перпендикулярны друг другу, а также направлению распространения волны.

Схема распространения электромагнитной волны

Опыт Майкелъсона — абсолютность скорости света

Физика того времени во многом строилась с учетом принципа относительности Галилея, согласно которому законы механики выглядят одинаково в любой выбранной инерциальной системе отсчета. В то же время согласно сложению скоростей – скорость распространения должна была зависеть от скорости движения источника. Однако, в таком случае электромагнитная волна вела бы себя по-разному в зависимости от выбора системы отсчета, что нарушает принцип относительности Галилея. Таким образом, вроде бы отлично сложенная теория Максвелла находилась в шатком состоянии.

Эксперименты показали, что скорость света действительно не зависит от скорости движения источника, а значит требуется теория, которая способна объяснить столь странный факт. Лучшей теорией на то время оказалась теория «эфира» — некой среды, в которой и распространяется свет, подобно тому как распространяется звук в воздухе. Тогда бы скорость света определялась бы не скоростью движения источника, а особенностями самой среды – эфира.

Предпринималось множество экспериментов с целью обнаружения эфира, наиболее известный из которых – опыт американского физика Альберта Майкелъсона. Говоря кратко, известно, что Земля движется в космическом пространстве. Тогда логично предположить, что также она движется и через эфир, так как полная привязанность эфира к Земле – не только высшая степень эгоизма, но и попросту не может быть чем-либо вызвана. Если Земля движется через некую среду, в которой распространяется свет, то логично предположить, что здесь имеет место сложение скоростей. То есть распространение света должно зависеть от направления движения Земли, которая летит через эфир. В результате своих экспериментов Майкелъсон не обнаружил какой-либо разницей между скоростью распространения света в обе стороны от Земли.

Эфирный ветер по отношению к Земле

Данную проблему попытался решить нидерландский физик Хендрик Лоренц. Согласно его предположению, «эфирный ветер» влиял на тела таким образом, что они сокращали свои размеры в направлении своего движения. Исходя из этого предположения, как Земля, так и прибор Майкелъсона, испытывали это Лоренцево сокращение, вследствие чего Альберт Майкелъсон получил одинаковую скорость для распространения света в обоих направлениях. И хотя Лоренцу несколько удалость оттянуть момент гибели теории эфира, все же ученые чувствовали, что данная теория «притянута за уши». Так эфир должен был обладать рядом «сказочных» свойств, в числе которых невесомость и отсутствие сопротивления движущимся телам.

Конец истории эфира пришел в 1905-м году вместе с публикацией статьи «К электродинамике движущихся тел» тогда еще мало известного – Альберта Эйнштейна.

Альберт Эйнштейн

Специальная теория относительности Альберта Эйнштейна

Двадцатишестилетний Альберт Эйнштейн высказывал совсем новый, иной взгляд на природу пространства и времени, который шел в разрез с тогдашними представлениями, и в особенности грубо нарушал принцип относительности Галилея. Согласно Эйнштейну, опыт Майкельсона не дал положительных результатов по той причине, что пространство и время имеют такие свойства, что скорость света есть абсолютная величина. То есть в какой бы системе отсчета не находился наблюдатель – скорость света относительно него всегда одна 300 000 км/сек. Из этого следовала невозможность применения сложения скоростей по отношению к свету – с какой бы скоростью не двигался источник света, скорость света не будет меняться (складываться или вычитаться).

Мысленный эксперимент с поездом. Абсолютность скорости света

Эйнштейн использовал Лоренцево сокращение для описания изменения параметров тел, движущихся со скоростями, близкими к скорости света. Так, например, длина таких тел будет сокращаться, а их собственное время – замедляться. Коэффициент таких изменений называется Лоренц-фактор. Известная формула Эйнштейна E=mc2на самом деле включает также Лоренц-фактор (E= ymc2), который в общем случае приравнивается к единице, в случае, когда скорость тела v равна нулю. С приближением скорости тела v к скорости света c Лоренц-фактор y устремляется к бесконечности. Из этого следует, что для того, чтобы разогнать тело до скорости света потребуется бесконечное количество энергии, а потому перейти этот предел скорости – невозможно.

В пользу данного утверждения существует также такой аргумент как «относительность одновременности».

Материалы по теме

Парадокс относительности одновременности СТО

Говоря кратко, явление относительности одновременности состоит в том, что часы, которые располагаются в разных точках пространства, могут идти «одновременно» только если они находятся в одной и той же инерциальной системе отсчета. То есть время на часах зависит от выбора системы отсчета.

Из этого же следует такой парадокс, что событие B, которое является следствием события A, может произойти одновременно с ним. Кроме того, можно выбрать системы отсчета таким образом, что событие B произойдет раньше, чем вызвавшее его событие A. Подобное явление нарушает принцип причинности, который довольно прочно укрепился в науке и ни разу не ставился под сомнение. Однако, данная гипотетическая ситуация наблюдается лишь в том случае, когда расстояние между событиями A и B больше, чем временной промежуток между ними, умноженный на «электромагнитную постоянную» — с. Таким образом, постоянная c, которой равна скорость света, является максимальной скоростью передачи информации. В противном бы случае нарушался бы принцип причинности.

Как измеряют скорость света?

 

Наблюдения Олаф Рёмера

Ученые античности в своем большинстве полагали, что свет движется с бесконечной скоростью, и первая оценка скорости света была получена аж в 1676-м году. Датский астроном Олаф Рёмер наблюдал за Юпитером и его спутниками. В момент, когда Земля и Юпитер оказались с противоположных сторон Солнца, затмение спутника Юпитера – Ио запаздывало на 22 минуты, по сравнению с рассчитанным временем. Единственное решение, которое нашел Олаф Рёмер – скорость света предельна. По этой причине информация о наблюдаемом событии запаздывает на 22 минуты, так как на прохождение расстояния от спутника Ио до телескопа астронома требуется некоторое время. Согласно подсчетам Рёмера скорость света составила 220 000 км/с.

Измерение скорости света Олафом Рёмером

Наблюдения Джеймса Брэдли

В 1727-м году английский астроном Джеймс Брэдли открыл явление аберрации света. Суть данного явления состоит в том, что при движении Земли вокруг Солнца, а также во время собственного вращения Земли наблюдается смещение звезд в ночном небе. Так как наблюдатель землянин и сама Земля постоянно меняют свое направление движения относительно наблюдаемой звезды, свет, излучаемый звездой, проходит различное расстояние и падает под разным углом к наблюдателю с течением времени. Ограниченность скорости света приводит к тому, что звезды на небосводе описывают эллипс в течение года. Данный эксперимент позволил Джеймсу Брэдли оценить скорость света — 308 000 км/с.

Звездная аберрация, обнаруженная Брэдли

Опыт Луи Физо

В 1849-м году французским физиком Луи Физо был поставлен лабораторный опыт по измерению скорости света. Физик установил зеркало в Париже на расстоянии 8 633 метров от источника, однако согласно расчетам Рёмера свет пройдет данное расстояние за стотысячные доли секунды. Подобная точность часов тогда была недостижима. Тогда Физо использовал зубчатое колесо, которое вращалось на пути от источника к зеркалу и от зеркала к наблюдателю, зубцы которого периодически закрывали свет. В случае, когда световой луч от источника к зеркалу проходил между зубцами, а на обратном пути попадал в зубец – физик увеличивал скорость вращения колеса вдвое. С увеличением скорости вращения колеса свет практически перестал пропадать, пока скорость вращения не дошла до 12,67 оборотов в секунду. В этот момент свет снова исчез.

Подобное наблюдение означало, что свет постоянно «натыкался» на зубцы и не успевал «проскочить» между ними. Зная скорость вращения колеса, количество зубцов и удвоенное расстояние от источника к зеркалу, Физо высчитал скорость света, которая оказалась равной 315 000 км/сек.

Схема опыта Луи Физо

Спустя год другой французский физик Леон Фуко провел похожий эксперимент, в котором вместо зубчатого колеса использовал вращающееся зеркало. Полученное ним значение скорости света в воздухе равнялось 298 000 км/с.

Спустя столетие метод Физо был усовершенствован настолько, что аналогичный эксперимент, поставленный в 1950-м году Э. Бергштрандом дал значение скорости равное 299 793,1 км/с. Данное число всего на 1 км/с расходится с нынешним значением скорости света.

Дальнейшие измерения

С возникновением лазеров и повышением точности измерительных приборов удалось снизить погрешность измерения вплоть до 1 м/с. Так в 1972-м году американские ученые использовали лазер для своих опытов. Измерив частоту и длину волны лазерного луча, им удалось получить значение – 299 792 458 м/с. Примечательно, что дальнейшее увеличение точности измерения скорости света в вакууме было нереализуемо в не в силу технического несовершенства инструментов, а из-за погрешности самого эталона метра. По этой причине в 1983-м году XVII Генеральная конференция по мерам и весам определила метр как расстояние, которое преодолевает свет в вакууме за время, равное 1 / 299 792 458 секунды.

Расстояние от Земли до Луны равняется 1,25 световых секунды

Подведем итоги

Итак, из всего вышесказанного следует, что скорость света в вакууме – фундаментальная физическая постоянная, которая фигурирует во многих фундаментальных теориях. Данная скорость абсолютна, то есть не зависит от выбора системы отсчета, а также равна предельной скорости передачи информации. С данной скоростью движутся не только электромагнитные волны (свет), но также и все безмассовые частицы. В том числе, предположительно, гравитон – частица гравитационных волн. Помимо всего прочего, в силу релятивистских эффектов собственное время для света буквально стоит.

Подобные свойства света, в особенности неприменимость к нему принципа сложения скоростей, не укладываются в голове. Однако, множество экспериментов подтверждают перечисленные выше свойства, и ряд фундаментальных теорий строятся именно на таковой природе света.

Понравилась запись? Расскажи о ней друзьям!

Просмотров записи: 23337

Запись опубликована: 18.06.2017
Автор: Владимир Соловьев

ЗАМЕДЛЕННЫЙ СВЕТ | Наука и жизнь

Понятие скорости распространения волны оказывается простым только в отсутствии дисперсии.

Лин Вестергард Хэу возле установки, на которой был проведен уникальный эксперимент.

Открыть в полном размере

Весной прошлого года научные и научно-популярные журналы мира сообщили сенсационную новость.

Американские физики провели уникальный эксперимент: они сумели понизить скорость света до 17 метров в секунду.

Все знают, что свет распространяется с огромной скоростью — почти 300 тысяч километров в секунду. Точное значение ее величины в вакууме = 299792458 м/с — фундаментальная физическая константа. Согласно теории относительности, это максимально возможная скорость передачи сигнала.

В любой прозрачной среде свет распространяется медленнее. Его скорость v зависит от показателя преломления среды n: v = с/n . Показатель преломления воздуха — 1,0003, воды — 1,33, различных сортов стекла — от 1,5 до 1,8. Одно из самых больших значений показателя преломления имеет алмаз — 2,42. Таким образом, скорость света в обычных веществах уменьшится не более чем в 2,5 раза.

В начале 1999 года группа физиков из Роуландовского института научных исследований при Гарвардском университете (штат Массачусетс, США) и из Стэнфордского университета (штат Калифорния) исследовала макроскопический квантовый эффект — так называемую самоиндуцированную прозрачность, пропуская лазерные импульсы через непрозрачную в обычных условиях среду. Этой средой были атомы натрия, находящиеся в особом состоянии, называемом бозе-эйнштейновским конденсатом. При облучении лазерным импульсом он приобретает оптические свойства, которые уменьшают групповую скорость импульса в 20 миллионов раз по сравнению со скоростью в вакууме. Экспериментаторам удалось довести скорость света до 17 м/с!

Прежде чем описывать сущность этого уникального эксперимента, напомним смысл некоторых физических понятий.

Групповая скорость. При распространении света в среде различают две скорости — фазовую и групповую. Фазовая скорость vф характеризует перемещение фазы идеальной монохроматической волны — бесконечной синусоиды строго одной частоты и определяет направление распространения света. Фазовой скорости в среде соответствует фазовый показатель преломления — тот самый, значения которого измеряются для различных веществ. Фазовый показатель преломления, а следовательно, и фазовая скорость зависят от длины волны. Эта зависимость называется дисперсией; она приводит, в частности, к разложению белого света, проходящего через призму, в спектр.

Но реальная световая волна состоит из набора волн различных частот, группирующихся в некотором спектральном интервале. Такой набор называют группой волн, волновым пакетом или световым импульсом. Эти волны распространяются в среде с различными фазовыми скоростями из-за дисперсии. При этом импульс растягивается, а его форма меняется. Поэтому для описания движения импульса, группы волн как целого, вводят понятие групповой скорости. Оно имеет смысл только в случае узкого спектра и в среде со слабой дисперсией, когда различие фазовых скоростей отдельных составляющих невелико. Для лучшего уяснения ситуации можно привести наглядную аналогию.

Представим себе, что на линии старта выстроились семь спортсменов, одетых в разноцветные майки по цветам спектра: красную, оранжевую, желтую и т. д. По сигналу стартового пистолета они одновременно начинают бег, но «красный» спортсмен бежит быстрее, чем «оранжевый», «оранжевый» — быстрее, чем «желтый», и т. д., так что они растягиваются в цепочку, длина которой непрерывно увеличивается. А теперь представим, что мы смотрим на них сверху с такой высоты, что отдельных бегунов не различаем, а видим просто пестрое пятно. Можно ли говорить о скорости движения этого пятна как целого? Можно, но только в том случае, если оно не очень расплывается, когда разница в скоростях разноцветных бегунов невелика. В противном случае пятно может растянуться на всю длину трассы, и вопрос о его скорости потеряет смысл. Это соответствует сильной дисперсии — большому разбросу скоростей. Если бегунов одеть в майки почти одного цвета, различающиеся лишь оттенками (скажем, от темно-красного до светло-красного), это станет соответствовать случаю узкого спектра. Тогда и скорости бегунов будут различаться ненамного, группа при движении останется достаточно компактной и может быть охарактеризована вполне определенной величиной скорости, которая и называется групповой.

Статистика Бозе-Эйнштейна. Это один из видов так называемой квантовой статистики — теории, описывающей состояние систем, содержащих очень большое число частиц, подчиняющихся законам квантовой механики.

Все частицы — как заключенные в атоме, так и свободные — делятся на два класса. Для одного из них справедлив принцип запрета Паули, в соответствии с которым на каждом энергетическом уровне не может находиться более одной частицы. Частицы этого класса называются фермионами (это электроны, протоны и нейтроны; в этот же класс входят частицы, состоящие из нечетного числа фермионов), а закон их распределения называется статистикой Ферми-Дирака. Частицы другого класса называются бозонами и не подчиняются принципу Паули: на одном энергетическом уровне может скапливаться неограниченное число бозонов. В этом случае говорят о статистике Бозе-Эйнштейна. К бозонам относятся фотоны, некоторые короткоживущие элементарные частицы (например, пи-мезоны), а также атомы, состоящие из четного числа фермионов. При очень низких температурах бозоны собираются на самом низком — основном — энергетическом уровне; тогда говорят, что происходит бозе-эйнштейновская конденсация. Атомы конденсата теряют свои индивидуальные свойства, и несколько миллионов их начинают вести себя как одно целое, их волновые функции сливаются, а поведение описывается одним уравнением. Это дает возможность говорить, что атомы конденсата стали когерентными, подобно фотонам в лазерном излучении. Исследователи из американского Национального института стандартов и технологий использовали это свойство конденсата Бозе-Эйнштейна для создания «атомного лазера» (см. «Наука и жизнь» № 10, 1997 г.).

Самоиндуцированная прозрачность. Это один из эффектов нелинейной оптики — оптики мощных световых полей. Он заключается в том, что очень короткий и мощный световой импульс проходит без ослабления через среду, которая поглощает непрерывное излучение или длинные импульсы: непрозрачная среда становится для него прозрачной. Самоиндуцированая прозрачность наблюдается в разреженных газах при длительности импульса порядка 10-7 — 10-8 с и в конденсированных средах — менее 10-11 c. При этом возникает запаздывание импульса — его групповая скорость сильно уменьшается. Впервые этот эффект был продемонстрирован Мак-Коллом и Ханом в 1967 году на рубине при температуре 4 К. В 1970 году в парах рубидия были получены задержки, соответствующие скоростям импульса, на три порядка (в 1000 раз) меньшим скорости света в вакууме.

Обратимся теперь к уникальному эксперименту 1999 года. Его осуществили Лен Вестергард Хэу, Захари Даттон, Сайрус Берузи (Роуландовский институт) и Стив Харрис (Стэнфордский университет). Они охладили плотное, удерживаемое магнитным полем облако атомов натрия до перехода их в основное состояние — на уровень с наименьшей энергией. При этом выделяли только те атомы, у которых магнитный дипольный момент был направлен противоположно направлению магнитного поля. Затем исследователи охладили облако до температуры менее 435 нК (нанокельвинов, т.е. 0,000000435 К, почти до абсолютного нуля).

После этого конденсат осветили «связующим пучком» линейно поляризованного лазерного света с частотой, соответствующей энергии его слабого возбуждения. Атомы перешли на более высокий энергетический уровень и перестали поглощать свет. В результате конденсат стал прозрачным для идущего следом лазерного излучения. И вот здесь появились очень странные и необычные эффекты. Измерения показали, что при определенных условиях импульс, проходящий через бозе-эйнштейновский конденсат, испытывает задержку, соответствующую замедлению света более чем на семь порядков — в 20 миллионов раз. Скорость светового импульса замедлилась до 17 м/с, а его длина уменьшилась в несколько раз — до 43 микрометров.

Исследователи считают, что, избежав лазерного нагрева конденсата, им удастся еще сильнее замедлить свет — возможно, до скорости нескольких сантиметров в секунду.

Система с такими необычными характеристиками позволит исследовать квантово-оптические свойства вещества, а также создавать различные устройства для квантовых компьютеров будущего, скажем, однофотонные переключатели.

Скорость света — сколько это, как измерить, способы достижения скорости света

Скорость света в вакууме — абсолютная величина скорости распространения электромагнитных волн в вакууме, постоянная и конечная. В физике традиционно обозначается латинской буквой «c». Скорость света в вакууме — фундаментальная постоянная, не зависящая от выбора инерциальной системы отсчёта (ИСО). Она относится к фундаментальным физическим постоянным, которые характеризуют не просто отдельные тела или поля, а свойства геометрии пространства-времени в целом. По современным представлениям, скорость света в вакууме — предельная скорость движения частиц и распространения взаимодействий равная 300 000 000 метров в секунду. Самое точно измерение на сегодняшний день показывает 299 792 458 ± 1,2 м/с. На сегодняшний день ученые всего мира пытаются создать космический корабль, который сможет приблизиться к скорости свете.

Самое обсуждаемое по теме Скорость света

Если квантовая теория верна, то от таких квантовых частиц как атомы, можно ожидать очень странного поведения. Но несмотря на хаос, коим может показаться квантовая физика, в этом удивительном мире крошечных частиц действуют свои собственные законы. Недавно команде ученых из Университета Бонна удалось доказать, что в квантовом мире – на уровне сложных квантовых операций – действует ограничение скорости. Атомы, будучи маленькими частицами, в некотором смысле напоминают пузырьки шампанского в бокале. Описать их можно как волны материи, однако их поведение больше напоминает бильярдный шар а не жидкость. Каждый, кому в голову придет идея очень быстро переместить атом из одного места в другое, должен действовать со знанием дела и сноровкой как у опытного официанта на банкете – не пролив ни капли шампанского из десятка бокалов на подносе, лавируя между столиками. Но даже в таком случае экспериментатор столкнется с определенным ограничением скорости – лимитом, превысить который невозможно. Полученные в ходе исследования результаты важны для работы квантовых компьютеров, а эта область, как наверняка знает уважаемый читатель, в последние годы активно развивается.

Читать далее

Если вы любите научную фантастику про космос, то наверняка знаете истории, где человечество путешествует по Вселенной на космических кораблях. Чтобы быстро перемещаться из одной точки необъятного космоса в другую, они оснащены варп-двигателями, которые позволяют достигать скоростей, превышающих скорость света (300 000 километров в секунду). К сожалению, на данный момент таких двигателей не существует. Но давайте представим, что они уже созданы и вы можете прямо сейчас отправиться в космическое путешествие? Допустим, у вас уже есть фантастический корабль и все, что вам остается — это запустить двигатель и отправиться в любую из понравившихся вам галактик. По словам представителей NASA, во время перемещения по космосу со скоростью света, у пилотов могут возникнуть серьезные проблемы. Чтобы рассказать о них, космическое агентство и художники представили мультфильм, в котором инопланетное существо отправляется в «космический отпуск». Получилось очень познавательно!

Читать далее

Межзвездные путешествия — непростая задача для будущих исследователей Вселенной. Из-за огромных расстояний, разделяющих звездные системы друг от друга, транспорту будущего придется научиться использовать неподвластные современному человеку физические силы. Но что, если все может оказаться гораздо проще, чем это считалось ранее? Как сообщает портал Universe Today, наши потомки смогут путешествовать между звездами на кораблях, движущихся сквозь время и пространство со скоростью света. Для того, чтобы претворить мечту человечества в реальность, транспортные средства будущего будут использовать гиперскоростные звезды и метеоры, ускоренные взрывами сверхновых. Так неужели мечта человечества о покорении Вселенной может однажды осуществиться?

Читать далее

Феномен замедления времени в космосе долгое время волновал умы писателей-фантастов со всего мира. Вместе с тем, вопрос о том, как сильно воздействует перемещение астронавта со скоростью света на его биологические часы, впервые был описан в так называемом “парадоксе близнецов”, в котором астронавт совершает путешествие в космос на скоростной ракете, а его брат-близнец остается на Земле. Считается, что по возвращению на голубую планету, астронавт обнаружит своего близнеца постаревшим, в то время как внешний вид самого космического путешественника останется едва ли не прежним.

Читать далее

Скорость света – это предел, с которым может двигаться материальный объект в пространстве, если, конечно, не брать в расчет гипотетические кротовые норы, с помощью которых, согласно предположениям, объекты могут перемещаться в пространстве еще быстрее. В идеальном вакууме частица света, фотон, может двигаться со скоростью 299 792 километра в секунду или примерно 1,079 миллиарда километров в час. На первый взгляд может показаться, что это удивительно быстро. Нет, это на самом деле быстро. Но в масштабах космоса такая скорость может быть мучительно медленной, особенно, если речь идет о радиосообщениях и полетах на другие планеты, в частности, находящиеся за пределами нашей Солнечной системы.

Читать далее

Скорость света является одной из важнейших констант в физике. Впервые оценку скорости света дал датский астроном Олаф Рёмер в 1676 году. Однако ученым, который установил, что именно свет задает верхний предел достижимой скорости в нашей Вселенной, равняющийся почти 300 000 километрам в секунду, был именно Альберт Эйнштейн. И все же, согласно той же теории Эйнштейна, все в этой Вселенной относительно, включая движение. Это, в свою очередь, заставляет задать вполне логичный вопрос: какова же скорость полной противоположности света – тьмы?

Читать далее

Когда варп-двигатель впервые был представлен людям — вместе с дебютом «Звездного пути» пятьдесят лет назад — наше понимание Вселенной принципиально отличалось от нынешнего. С одной стороны, варп-двигатель был просто сюжетным устройством, которое позволяло добираться до далеких звезд весьма быстро; казалось, он нарушает принцип относительности Эйнштейна и физически невозможен. С другой стороны, казалось, что гравитация стягивает далекие галактики между собой, и если двигаться достаточно близко к скорости света, можно достичь чего угодно. Тогда мы не знали о темной энергии.

Читать далее

Группа ученых из МГУ имени М. В. Ломоносова и Технологического университета Тойохаши (что в Японии) разработала метод управления поворотом поляризации света. Как утверждают физики, это сможет открыть новые горизонты для развития систем оптической обработки информации, а также поможет в создании нового типа сверхбыстрых компьютеров, в которых вместо электронов работают фотоны.

Читать далее

Если посмотреть на Солнце через 150 миллионов километров космоса, который разделяет наш мир от ближайшей звезды, свет, который вы видите, не показывает Солнце на текущий момент, а каким оно было 8 минут и 20 секунд назад. Это потому что свет движется не мгновенно (а со скоростью света, хаха): его скорость составляет 299 792,458 километра в секунду (подробности этого невероятного факта здесь). Именно такое время нужно свету, чтобы преодолеть путь от фотосферы Солнца до нашей планеты. Но силе тяжести не обязательно нужно вести себя так же; возможно, как предсказывала теория Ньютона, гравитационная сила представляет собой мгновенное явление и ощущается всеми объектами с массой во Вселенной, через все эти огромные космические расстояния, одновременно.

Читать далее

Исследователи из Гарвардского университета заявили о создании способа управления светом на наноуровне, который может привести к созданию фотонных телекоммуникаций (вместо современных электронных). Как сообщается, команда исследователей разработала метаматериал из кремниевых опор, заключённых в полимер и обёрнутых золотой плёнкой, которая снижает коэффициент преломления до нуля. Говоря русским языком, это означает, что световая волна может проходить этот материал со скоростью, стремящейся к бесконечности, при этом не нарушая известные законы физики.

Читать далее

почему скорость света такая, какая есть? / Хабр

Вне зависимости от цвета, длины волны или энергии, скорость, с которой свет перемещается в вакууме, остаётся постоянной. Она не зависит от местоположения или направлений в пространстве и времени

Ничто во Вселенной не способно двигаться быстрее света в вакууме. 299 792 458 метров в секунду. Если это массивная частица, она может лишь приблизиться к этой скорости, но не достичь её; если это безмассовая частица, она всегда должна двигаться именно с этой скоростью, если дело происходит в пустом пространстве. Но откуда нам это известно и что тому причиной? На этой неделе наш читатель задаёт нам три связанных со скоростью света вопроса:

Почему скорость света конечна? Почему она именно такая, какая есть? Почему не быстрее и не медленнее?

Вплоть до XIX века у нас даже не было подтверждений этим данным.


Иллюстрация света, проходящего через призму и разделяющегося на чёткие цвета.

Если свет проходит через воду, призму или любую другую среду, он разделяется на разные цвета. Красный цвет преломляется не под тем углом, под которым это делает синий, из-за чего и возникает что-то типа радуги. Это можно наблюдать и вне видимого спектра; инфракрасный и ультрафиолетовый свет ведут себя так же. Это было бы возможно, только если скорость света в среде отличается для света разных длин волн/энергий. Но в вакууме, вне всякой среды, всякий свет перемещается с одной и той же конечной скоростью.


Разделение света на цвета происходит из-за разных скоростей движения света, зависящих от длины волны, через среду

До этого додумались только в середине XIX века, когда физик Джеймс Клерк Максвелл показал, что на самом деле представляет собой свет: электромагнитную волну. Максвелл впервые поставил независимые явления электростатики (статичные заряды), электродинамики (движущиеся заряды и токи), магнитостатики (постоянные магнитные поля) и магнитодинамики (наведённые токи и переменные магнитные поля) на единую, объединённую платформу. Управляющие ею уравнения – уравнения Максвелла – позволяют вычислять ответ на простой вроде бы вопрос: какие типы электрических и магнитных полей могут существовать в пустом пространстве вне электрических или магнитных источников? Без зарядов и без токов можно было бы решить, что никакие – но уравнения Максвелла удивительным образом доказывают обратное.


Табличка с уравнениями Максвелла с обратной стороны его памятника

Ничто – одно из возможных решений; но возможно и другое – колеблющиеся в одной фазе взаимно перпендикулярные электрическое и магнитное поля. У них есть определённые амплитуды. Их энергия определяется частотой колебаний полей. Они передвигаются с определённой скоростью, определяемой двумя константами: ε0 и µ0. Эти константы определяют величину электрического и магнитного взаимодействий в нашей Вселенной. Получаемое уравнение описывает волну. И, как у всякой волны, у неё есть скорость, 1/√ε0 µ0, которая оказывается равной c, скорости света в вакууме.


Колеблющиеся в одной фазе взаимно перпендикулярные электрическое и магнитное поля, распространяющиеся со скоростью света, определяют электромагнитное излучение

С теоретической точки зрения, свет – безмассовое электромагнитное излучение. По законам электромагнетизма он обязан двигаться со скоростью 1/√ε0 µ0, равной c – вне зависимости от остальных его свойств (энергии, импульса, длины волны). ε0 можно измерить, сделав и измерив конденсатор; µ0 точно определяется из ампера, единицы электрического тока, что и даёт нам c. Та же фундаментальная константа, впервые выведенная Максвеллом в 1865 году, с тех пор появлялась во многих других местах:

• Это скорость любой безмассовой частицы или волны, включая гравитационные.
• Это фундаментальная константа, соотносящая ваше движение в пространстве с вашим движением во времени в теории относительности.
• И это фундаментальная константа, связывающая энергию с массой покоя, E = mc2


Наблюдения Рёмера снабдили нас первыми измерениями скорости света, полученными при помощи геометрии и измерения времени, необходимого на то, чтобы свет прошёл расстояние, равное диаметру орбиты Земли.

Первые измерения этой величины были сделаны во время астрономических наблюдений. Когда луны Юпитера входят и выходят в положение затмения, они кажутся видимыми или невидимыми с Земли в определённой последовательности, зависящей от скорости света. Это привело к первому количественному измерению с в XVII веке, которое определили в 2,2 × 108 м/с. Отклонение звёздного света – из-за движения звезды и Земли, на которой установлен телескоп – тоже можно оценить численно. В 1729 году этот метод измерения с показал значение, отличающееся от современного всего на 1,4%. К 1970-м с определили равным 299 792 458 м/с с погрешностью всего в 0,0000002%, большая часть которой проистекала из невозможности точного определения метра или секунды. К 1983 году секунду и метр переопределили через с и универсальные свойства излучения атома. Теперь скорость света равна точно 299 792 458 м/с.


Атомный переход с орбитали 6S, δf1, определяет метр, секунду и скорость света

Так почему же скорость света не больше и не меньше? Объяснение такое же простое, как указанный на рис. Выше атом. Атомные переходы происходят так, как происходят, из-за фундаментальных квантовых свойств строительных блоков природы. Взаимодействия атомного ядра с электрическим и магнитными полями, создаваемыми электронами и другими частями атома приводят к тому, что разные энергетические уровни оказываются чрезвычайно близко друг к другу, но всё же немного отличаются: это называется сверхтонким расщеплением. В частности, частота перехода сверхтонкой структуры цезия-133 испускает свет совершенно определённой частоты. Время, за которое проходит 9 192 631 770 таких циклов, определяет секунду; расстояние, которое свет проходит за это время, равняется 299 792 458 метрам; скорость, с которой распространяется этот свет, определяет с.


Пурпурный фотон переносит в миллион раз больше энергии, чем жёлтый. Космический гамма-телескоп Ферми не показывает никаких задержек какого-либо из фотонов, пришедших к нам от гамма-всплеска, что подтверждает постоянство скорости света для всяких энергий

Чтобы поменять это определение, нужно, чтобы с этим атомным переходом или с идущим от него светом произошло что-то фундаментально отличное от его текущей природы. Этот пример также даёт нам ценный урок: если бы атомная физика и атомные переходы работали бы в прошлом или на дальних расстояниях по-другому, это было бы свидетельством изменения скорости света со временем. Пока что все проводимые нами измерения лишь накладывают дополнительные ограничения на постоянство скорости света, и эти ограничения весьма строги: изменение не превосходит 7% от текущего значения за последние 13,7 млрд лет. Если бы по какой-то из этих метрик скорость света оказалась не постоянной, или же она отличалась бы у разных типов света, это привело бы к крупнейшей научной революции со времён Эйнштейна. Вместо этого все свидетельства говорят в пользу Вселенной, в которой все законы физики всегда, везде, во всех направлениях, во все времена остаются одинаковыми, включая и физику самого света. В каком-то смысле это тоже достаточно революционные сведения.

Итан Сигель – астрофизик, популяризатор науки, автор блога Starts With A Bang! Написал книги «За пределами галактики» [Beyond The Galaxy], и «Трекнология: наука Звёздного пути» [Treknology].

5.1.1: Скорости различных типов волн

  1. Последнее обновление
  2. Сохранить как PDF
  • Идентификатор страницы
    26167
    • Кайл Форинаш и Вольфганг Кристиан

    Скорость волны определяется типом волны и физическими свойствами среды, в которой она распространяется. Исключение составляют электромагнитные волны, которые могут распространяться в вакууме. Для большинства веществ материал будет вибрировать, подчиняясь силе закона Гука, когда волна проходит через него, и скорость не будет зависеть от частоты. Электромагнитные волны в вакууме и волны, распространяющиеся в линейной среде, называются линейными волнами и имеют постоянную скорость. Примеры: 9{\ circ} \ text {C} \}} Вещество Скорость звука (\(\text{м/с}\)) Углекислый газ \(259\) Водород \(1284\) Гелий \(965\) 9{\ circ} \ text {C} \}} Глицерин \(1904\) Морская вода (\(3,5\)% солености) \(1535\) Вода \(1493\) Меркурий \(1450\) Керосин \(1324\) Метиловый спирт \(1103\) Четырёххлористый углерод \(926\) Твердые вещества Алмаз \(12000\) Стекло пирекс \(5640\) Железо \(5960\) Гранит \(6000\) Алюминий \(5100\) Латунь \(4700\) Медь (отожженная) \(4760\) Золото \(3240\) Свинец (отожженный) \(2160\) Резина (резинка) \(1550\)

    Таблица \(\PageIndex{1}\)

    Вот более полный список скорости звука в различных материалах.

    Как мы видели в предыдущей главе, существует зависимость между периодом, длиной волны и скоростью волны. На период плавания пробки в воде влияет скорость прохождения волны (скорость волны) и расстояние между вершинами (длина волны). Соотношение между скоростью, периодом и длиной волны синусоидальной волны задается формулой \(v=\lambda /T\) , где длина волны и период синусоидальной волны были определены ранее. Это также можно записать как \(v=\lambda f\), поскольку частота является обратной величиной периода и верно для всех линейных волн. Обратите внимание, что, поскольку скорость волны обычно является фиксированной величиной, частота и длина волны будут обратно пропорциональны; более высокие частоты означают более короткие длины волн.

    Часто проще написать \(ω = 2πf\), где \(\omega\) угловая частота в радианах в секунду, чем везде писать \(2\pi f\). Точно так же проще написать \(k=2\pi /\lambda \), где \(k\) — волновое число в радианах на метр, чем писать \(2\pi /\ лямбда\) много. {\circ}\text{C}\)) звук распространяется со скоростью \(344\text{м/с}\). 9{8}\text{ м/с}\) в вакууме, но замедляются при прохождении через среду (например, при переходе света из воздуха в стекло). Это происходит потому, что материал имеет другое значение диэлектрической и/или магнитной проницаемости из-за взаимодействия волны с атомами материала. Величина изменения скорости определяется показателем преломления \(n=c/v\), где \(c\) — скорость света в вакууме, а \(v\) — скорость в среде. Частота волны не меняется при ее замедлении, так как \(v=\lambda f\), длина волны электромагнитных волн в среде должна быть несколько меньше.

    Примеры видео/аудио:

    • Какова скорость звука в вакууме? Зуммер в стеклянном стекле. Почему нет звука при удалении воздуха из банки?
    • Демонстрация скорости звука в различных газах. Почему нет звука при удалении воздуха из банки?
    • Эти два видеоролика демонстрируют эффект Allasonic. Скорость звука в жидкости с пузырьками воздуха разная, потому что разная плотность. По мере того как пузырьки лопаются, скорость звука изменяется, что приводит к изменению частоты звуковых волн в столбе жидкости и, таким образом, к изменению высоты тона. Пример: раз, два. Что вы слышите в каждом случае?
    • Зубная трубка — это игрушка, внутри которой находится пружина, прикрепленная к двум пластиковым чашкам на каждом конце. Вибрации пружины распространяются с разной скоростью, поэтому звук, начинающийся на одном конце (например, щелчок, когда вы встряхиваете трубку, и пружина ударяет по чашке), в конечном итоге меняет высоту звука на другом конце по мере поступления различных частот. Другими словами, это нелинейная система. Посмотрите, сможете ли вы понять из видео, какие частоты распространяются быстрее, высокие частоты или низкие.

    {8}\text{ м/с}\), но звуковые волны распространяются со скоростью примерно \(344\text{ м/с}\). Каково время задержки света и звука от источника, который находится на расстоянии \(10 000\text{м}\) (это можно использовать для получения приблизительного расстояния до грозы)?

  • Какие две ошибки допускают в научно-фантастических фильмах, когда вы одновременно видите и слышите взрыв в космосе?
  • См. таблицу скорости звука в различных веществах. Если вы держите одно ухо в воде, а одно высовываете во время купания в озере, и звонит колокол, который находится наполовину в воде на некотором расстоянии от вас, какое ухо первым услышит звук? 9{\ circ} \ text {C} \}}?
  • Как бы звучал оркестр, если бы разные инструменты издавали звуки, распространяющиеся с разной скоростью?
  • Скорость волны определяется средой, в которой она распространяется, поэтому в данной ситуации она обычно постоянна. Что произойдет с частотой волны, если длину волны увеличить вдвое?
  • Что произойдет с длиной волны, если частоту увеличить вдвое, а скорость останется той же?
  • Предположим, что звуковая волна имеет частоту \(200\text{ Гц}\). Если скорость звука \(343\text{ м/с}\), какой длины волны эта волна?
  • Какие факторы определяют скорость звука в воздухе?
  • Почему звуковые волны распространяются быстрее в жидкостях, чем в воздухе?
  • Почему звуковые волны распространяются быстрее через твердые тела, чем через жидкости?
  • Скорость звука в жидкости определяется выражением \(v=\sqrt{B/Q}\), где \(B\) — объемный модуль (сжимаемость), а \(Q\) — плотность. Что произойдет со скоростью, если плотность жидкости увеличится?
  • Что должно быть верно в отношении сжимаемости \(B\) воды по сравнению с воздухом, учитывая, что звук в воде распространяется быстрее, а вода плотнее воздуха?
  • Скорость звука в жидкости определяется выражением \(v=\sqrt{B/Q}\), где \(B\) — объемный модуль (сжимаемость), а \(Q\) — плотность. Можете ли вы придумать умный способ измерения объемного модуля жидкости, если у вас есть простой способ измерить скорость звука в жидкости? Объяснять.
  • Скорость звука на струне определяется выражением \(v=\sqrt{T/\mu}\), где \(T\) — натяжение в ньютонах, а \(\mu\) — линейная плотность (толщина) в \(\text{кг/м}\). Вы также знаете, что \(v=f\lambda\). Приведите два способа изменения частоты колебаний гитарной струны, основанные на знании этих двух уравнений.
  • Что касается предыдущего вопроса, увеличение напряжения влияет на частоту? Что делает использование более плотной струны с частотой?
  • На следующем графике изображена волна, застывшая во времени в точке \(t = 0\). Уравнение, описывающее волну, имеет вид \(y(x,t)=A\cos (kx-\omega t+\phi )\). Нарисуйте эффект удвоения амплитуды \(A\).
  • Рисунок \(\PageIndex{1}\)

    1. Для следующего графика волны нарисуйте эффект удвоения длины волны.

    Рисунок \(\PageIndex{2}\)

    1. Математическое описание синусоиды дается выражением \(y(x,t)=A\cos (kx-\omega t+\phi )\) . Объясните, что представляет каждый из терминов \((A, k, \omega, \phi )\).

    Эта страница под названием 5.1.1: Speeds of Different Types of Waves распространяется под лицензией CC BY-NC-SA 3.0 и была создана, изменена и/или курирована Кайлом Форинашем и Вольфгангом Кристианом через исходный контент, который был отредактирован для стиль и стандарты платформы LibreTexts; подробная история редактирования доступна по запросу.

    1. Наверх
      • Была ли эта статья полезной?
      1. Тип изделия
        Раздел или Страница
        Автор
        Кайл Форинаш и Вольфганг Кристиан
        Лицензия
        СС BY-NC-SA
        Версия лицензии
        3,0
      2. Теги
        1. источник@https://www. compadre.org/books/SoundBook

      специальная теория относительности — Почему скорости разных электромагнитных волн в вакууме НЕ ТОЧНО равны?

      Спросил

      Изменено 2 года, 11 месяцев назад

      Просмотрено 171 раз

      $\begingroup$

      В моем учебнике (ссылка ниже) сказано, что разные волны электромагнитного спектра имеют скорости , почти равные друг другу. (Вариации в пределах нескольких м/с по моему учебнику). Точная формулировка в книге:

      .

      Скорость электромагнитных волн в свободном пространстве или вакууме важная фундаментальная постоянная. Это показали опыты на электромагнитных волн различных длин волн, что эта скорость то же (независимо от длины волны) с точностью до нескольких метров в секунду, вне величиной $3 \times 10^8$ м/с.

      Почему не равно ?

      Не из-за отсутствия ли таких высокоточных измерительных приборов, которые еще только предстоит открыть? Если нет, то в чем причина? Просьба уточнить.

      Кроме того, возможно ли, чтобы другая электромагнитная волна двигалась быстрее скорости света? Логически спрашивая, почему бы и нет?

      Книга NCERT STD 12th Physics, глава 8 раздел 8.3.2. стр.276-277. ссылка на PDF главы 8.

      • специальная теория относительности
      • электромагнитное излучение
      • скорость света
      • измерения
      • анализ ошибок

      $\endgroup$

      4

      $\begingroup$

      Почему не точно равны? Связано ли это с отсутствием таких высокоточных измерительных приборов, которые еще только предстоит открыть?

      Ни один эксперимент не может доказать, что две величины точно равны. Все, что может сделать любой эксперимент, — это определить, что разница между двумя величинами меньше экспериментальной точности. Более точные устройства, как вы упомянули, могут уменьшить этот диапазон, но никогда не могут утверждать, что они точно равны.

      Формулировка в учебнике является хорошим примером того, как правильно описать результат эксперимента, который можно было бы вольно интерпретировать как доказательство того, что они равны.

      $\endgroup$

      $\begingroup$

      Вы правы, говоря, что разница между скоростью света и электромагнитных волн связана с ошибкой измерения. Вы ошибаетесь, полагая, что эта ошибка все еще актуальна.

      В вашем учебнике написано:

      98 м/с), полученное из оптических измерений.

      Здесь они говорят, что во времена Максвелла измерения скорости света дали значение приблизительно 300 000 км/с. Уравнения Максвелла (с использованием принятых в то время значений диэлектрической и магнитной проницаемости вакуума) давали его электромагнитным волнам скорость 299 800 км/с. В результате Максвелл предположил, что свет — это электромагнитная волна, а незначительная разница — просто ошибка измерения.

      Поскольку теперь мы знаем, что свет действительно является электромагнитной волной (как и радио, рентгеновские лучи и т. д.), мы знаем, что все они распространяются с одной и той же скоростью c, которая, как теперь известно, равна , ровно 299 792,458 км/с . Причина, по которой это значение является точным, заключается в том, что метр определяется как расстояние, пройденное светом (или другими электромагнитными волнами) в вакууме за $\frac1{299,792,458}$ секунды.

      $\endgroup$

      $\begingroup$

      Возможно, имеет место погрешность измерения длины волны, особенно для низкочастотных электромагнитных волн, таких как радиоволны. То есть, когда одиночная радиоволна излучается, например, антенной, приемник (например, детектор волнового пика), расположенный далеко от излучателя, может активироваться как принимающая точка, близкая к 98 \пространство м/с$

      $\endgroup$

      Твой ответ

      Зарегистрируйтесь или войдите в систему

      Зарегистрируйтесь с помощью Google

      Зарегистрироваться через Facebook

      Зарегистрируйтесь, используя адрес электронной почты и пароль

      Опубликовать как гость

      Электронная почта

      Требуется, но не отображается

      Опубликовать как гость

      Электронная почта

      Требуется, но не отображается

      Нажимая «Опубликовать свой ответ», вы соглашаетесь с нашими условиями обслуживания, политикой конфиденциальности и политикой использования файлов cookie

      .

      Скорость света | Что такое скорость света?

      Где-то в открытом космосе, в миллиардах световых лет от Земли, изначальный свет, связанный с Большим взрывом Вселенной, прожигает новую землю, продолжая свое движение наружу. В отличие от другой формы электромагнитного излучения, происходящего на Земле, радиоволны из первого живого эпизода Шоу Люси транслируют премьеру где-то в глубоком космосе, хотя и значительно уменьшенной по амплитуде.

      В основе обоих событий лежит понятие скорости света (и всех других форм электромагнитного излучения).

      Насколько высока скорость света

      Скорость света, тщательно изученная учеными, теперь выражается постоянной величиной, обозначаемой в уравнениях символом c. Не совсем константа, а скорее максимальная скорость в вакууме, скорость света в км, что составляет почти 300 000 километров в секунду, которой можно манипулировать путем изменения среды или с помощью квантовой интерференции.

      Свет, распространяющийся в однородном веществе или среде, распространяется прямолинейно с относительно постоянной скоростью, если только он не преломляется, не отражается, не дифрагирует или не возмущается каким-либо другим образом. Этот общепризнанный научный факт не является продуктом атомной эры или даже эпохи Возрождения, а первоначально продвигался древнегреческим ученым Евклидом где-то около 350 г. до н.э. в его эпохальном трактате «Оптика». Однако интенсивность света (и другого электромагнитного излучения) обратно пропорциональна квадрату пройденного расстояния. Таким образом, после того как свет дважды прошел заданное расстояние, его интенсивность падает в четыре раза.

      Какова скорость света в воздухе и воде?

      Когда свет, путешествуя по воздуху, попадает в другую среду, например стекло или воду, скорость и длина волны света уменьшаются (см. рис. 2), хотя частота остается неизменной. Свет распространяется со скоростью примерно 300 000 километров в секунду в вакууме, который имеет показатель преломления 1,0, но замедляется до 225 000 километров в секунду в воде (показатель преломления 1,3; см. рис. 2) и 200 000 километров в секунду в стекле (показатель преломления индекс 1,5). В алмазе с довольно высоким показателем преломления 2,4 скорость света снижена до относительной малой скорости (125 000 километров в секунду), что примерно на 60 процентов меньше его максимальной скорости в вакууме.

      Из-за огромных перемещений света в космическом пространстве между галактиками (см. рис. 1) и внутри Млечного Пути расстояние между звездами измеряется не километрами, а световыми годами, расстояние, которое свет проходит за год . Световой год равен 9,5 трлн километров или около 5,9 трлн миль. Расстояние от Земли до ближайшей за нашим Солнцем звезды, Проксимы Центавра, составляет примерно 4,24 световых года. Для сравнения, диаметр галактики Млечный Путь оценивается примерно в 150 000 световых лет, а расстояние до галактики Андромеды составляет примерно 2,21 миллиона световых лет. Это означает, что свет, покинувший галактику Андромеды 2,21 миллиона лет назад, только что достиг Земли, если только он не был задержан отражающими небесными телами или преломляющими осколками.

      Когда астрономы смотрят в ночное небо, они наблюдают смесь реального времени, недавнего прошлого и древней истории. Например, в период, когда первые вавилоняне, арабские астрологи и греческие астрономы описывали звездные созвездия, Скорпион (Скорпион для астрологов) все еще имел хлыст скорпиона. Хвостовая звезда и другие звезды этого созвездия появились в небе в виде новых звезд между 500 и 1000 годами до нашей эры, но сегодняшним астрономам они больше не видны. Хотя некоторые из звезд, наблюдаемых на ночном небе Земли, давно погибли, световые волны, несущие их изображения, все еще достигают человеческих глаз и телескопов. По сути, свет от их разрушения (и тьма их отсутствия) еще не преодолели огромные расстояния глубокого космоса из-за нехватки времени.

      Ранняя история скорости света

      Эмпедокл из Акрагаса, живший около 450 г. до н.э., был одним из первых зарегистрированных философов, предположивших, что свет распространяется с конечной скоростью. Почти тысячелетие спустя, примерно в 525 году нашей эры, римский ученый и математик Аниций Боэций попытался задокументировать скорость света, но после того, как его обвинили в измене и колдовстве, он был обезглавлен за свои научные усилия. С тех пор, как китайцы впервые применили черный порох для фейерверков и сигналов, человек задавался вопросом о скорости света. Когда вспышка света и цвета предшествовала взрывному звуку на несколько секунд, не требовалось серьезных расчетов, чтобы понять, что скорость света явно превышает скорость звука.

      Скорость света в прозрачных материалах

      Узнайте, как скорость света уменьшается пропорционально показателю преломления материала, когда свет попадает в новую прозрачную среду, такую ​​как воздух, вода или стекло.

      Начало обучения

      Китайские секреты взрывчатых веществ дошли до Запада в середине тринадцатого века, а вместе с ними пришли и вопросы о скорости света. До этого периода другие исследователи, должно быть, считали вспышку молнии, за которой следует раскат грома, типичной для грозы, но не предлагали правдоподобных научных объяснений природы задержки. Арабский ученый Альхазен был первым серьезным ученым-оптиком, предположившим (около 1000 г. н.э. ), что свет имеет конечную скорость, а к 1250 г. н.э. пионер британской оптики Роджер Бэкон писал, что скорость света конечна, хотя и очень высока. Тем не менее, широко распространенное мнение большинства ученых в этот период состояло в том, что скорость света бесконечна и не может быть измерена.

      В 1572 году знаменитый датский астроном Тихо Браге первым описал сверхновую, которая произошла в созвездии Кассиопеи. Увидев, как «новая звезда» внезапно появляется на небе, постепенно увеличивая яркость, а затем исчезая из поля зрения в течение 18 месяцев, астроном был озадачен, но заинтригован. Эти новые небесные видения заставили Браге и его современников усомниться в широко распространенном представлении о совершенной и неизменной Вселенной, имеющей бесконечную скорость света. Веру в то, что свет имеет бесконечную скорость, было трудно опровергнуть, хотя некоторые ученые начали сомневаться в скорости света еще в шестнадцатом веке. Еще в 1604 году немецкий физик Иоганн Кеплер предположил, что скорость света мгновенна. В своих опубликованных заметках он добавлял, что космический вакуум не замедлял скорость света, в какой-то степени затрудняя поиски его современниками эфира, якобы заполняющего пространство и несущего свет.

      Оценки скорости света Оле Ремера

      Вскоре после изобретения и некоторых относительно грубых усовершенствований телескопа датский астроном Оле Ремер (в 1676 г.) был первым ученым, предпринявшим строгую попытку оценить скорость света. Изучая спутник Юпитера Ио и его частые затмения, Ремер смог предсказать периодичность периода затмения для Луны (рис. 3). Однако через несколько месяцев он заметил, что его предсказания постепенно становятся менее точными из-за постепенно увеличивающихся временных интервалов, достигая максимальной ошибки около 22 минут (довольно большое расхождение, учитывая, как далеко свет проходит за этот промежуток времени). Затем, как ни странно, его предсказания снова стали более точными в течение нескольких месяцев, и цикл повторился. Работая в Парижской обсерватории, Ремер вскоре понял, что наблюдаемые различия были вызваны различиями в расстоянии между Землей и Юпитером из-за орбитальных путей планет. По мере того, как Юпитер удалялся от Земли, свету приходилось преодолевать большее расстояние, и для достижения Земли требовалось дополнительное время. Применяя относительно неточные расчеты расстояний между Землей и Юпитером, доступные в тот период, Ремер смог оценить скорость света примерно в 137 000 миль (или 220 000 километров) в секунду. Рисунок 3 иллюстрирует репродукцию оригинальных рисунков Ремера, иллюстрирующих его методологию, используемую для определения скорости света.

      Работа Ремера всколыхнула научное сообщество, и многие исследователи начали пересматривать свои предположения о бесконечной скорости света. Сэр Исаак Ньютон, например, писал в своем знаменательном трактате 1687 года «Philosophiae Naturalis Prinicipia Mathematica» («Математические принципы натуральной философии»): «Ибо теперь из явлений спутников Юпитера, подтвержденных наблюдениями различных астрономов, ясно, что свет распространяется последовательно, и требуется около семи или восьми минут, чтобы добраться от Солнца до Земли», что на самом деле является удивительно точной оценкой правильной скорости света. Уважаемое мнение Ньютона и его широкая репутация сыграли важную роль в начале научной революции и помогли начать новые исследования ученых, которые теперь признали конечную скорость света.

      Оценки скорости света Джеймса Брэдли

      Следующим, кто дал полезную оценку скорости света, был британский физик Джеймс Брэдли. В 1728 году, через год после смерти Ньютона, Брэдли оценил скорость света в вакууме примерно в 301 000 километров в секунду, используя звездные аберрации. Эти явления проявляются в кажущемся изменении положения звезд из-за движения Земли вокруг Солнца. Степень звездной аберрации можно определить по отношению орбитальной скорости Земли к скорости света. Измерив угол звездной аберрации и применив эти данные к орбитальной скорости Земли, Брэдли смог получить удивительно точную оценку.

      В 1834 году сэр Чарльз Уитстон, изобретатель калейдоскопа и пионер науки о звуке, попытался измерить скорость электричества. Уитстон изобрел устройство, в котором использовались вращающиеся зеркала и емкостной разряд через лейденскую банку, чтобы генерировать и синхронизировать движение искр по проводу почти в восемь миль. К сожалению, его расчеты (и, возможно, его приборы) были ошибочны до такой степени, что Уитстон оценил скорость электричества в 288 000 миль в секунду, ошибка, которая привела его к мысли, что электричество движется быстрее света. Позднее исследования Уитстона были расширены французским ученым Домиником Франсуа Жаном Араго. Хотя ему не удалось завершить свою работу до того, как в 1850 году у него ухудшилось зрение, Араго правильно постулировал, что свет в воде распространяется медленнее, чем в воздухе.

      Эксперименты Физо и Фуко со скоростью света

      Тем временем во Франции конкурирующие ученые Арманд Физо и Жан-Бернар-Леон Фуко независимо друг от друга попытались измерить скорость света, не полагаясь на небесные явления, воспользовавшись открытиями Араго и расширив конструкцию прибора с вращающимся зеркалом Уитстона. В 1849 году Физо сконструировал устройство, которое направляло луч света через зубчатое колесо (вместо вращающегося зеркала), а затем на неподвижное зеркало, расположенное на расстоянии 5,5 миль. Вращая колесо с большой скоростью, он смог направить луч через зазор между двумя зубьями на пути наружу и поймать отраженные лучи в соседнем зазоре на обратном пути. Вооружившись скоростью вращения колеса и расстоянием, пройденным импульсным светом, Физо смог рассчитать скорость света. Он также обнаружил, что свет распространяется быстрее в воздухе, чем в воде (подтверждая гипотезу Араго), факт, который его соотечественник Фуко позже подтвердил экспериментально.

      Фуко использовал быстро вращающееся зеркало, приводимое в движение турбиной со сжатым воздухом, для измерения скорости света. В его аппарате (см. рис. 4) узкий пучок света проходит через отверстие, а затем через стеклянное окно (действующее также как светоделитель) с мелкой шкалой, после чего попадает на быстро вращающееся зеркало. Свет, отраженный от вращающегося зеркала, направляется через батарею стационарных зеркал по зигзагообразной схеме, предназначенной для увеличения длины пути прибора примерно до 20 метров без соответствующего увеличения размеров. За то время, которое потребовалось свету, чтобы отразиться от ряда зеркал и вернуться к вращающемуся зеркалу, произошло небольшое смещение положения зеркала. Впоследствии свет, отраженный от смещенного положения вращающегося зеркала, следует по новому пути обратно к источнику и в микроскоп, установленный на приборе. Крошечное смещение света можно было увидеть в микроскоп и записать. Анализируя данные своего эксперимента, Фуко смог рассчитать скорость света как 298 000 километров в секунду (примерно 185 000 миль в секунду).

      Путь света в устройстве Фуко был достаточно коротким, чтобы его можно было использовать для измерения скорости света в других средах, кроме воздуха. Он обнаружил, что скорость света в воде или стекле составляет лишь около двух третей скорости света в воздухе, а также пришел к выводу, что скорость света в данной среде обратно пропорциональна показателю преломления. Этот замечательный результат согласуется с предсказаниями о поведении света, сделанными сотнями лет назад на основе волновой теории распространения света.

      Прибор скорости света Майкельсона и Морли

      Следуя примеру Фуко, американский физик польского происхождения по имени Альберт А. Майкельсон попытался повысить точность метода и успешно измерил скорость света в 1878 году с помощью более сложной версии прибора вдоль стены высотой 2000 футов. берегу реки Северн в Англии. Вложив средства в высококачественные линзы и зеркала для фокусировки и отражения луча света на гораздо более длинном пути, чем тот, который использовал Фуко, Майкельсон рассчитал окончательный результат в 186 355 миль в секунду (299 909 километров в секунду), допуская возможную ошибку около 30 миль в секунду. Из-за повышенной сложности его экспериментального плана точность измерения Майкельсона была более чем в 20 раз выше, чем у Фуко.

      В конце 1800-х годов большинство ученых все еще считали, что свет распространяется в пространстве с использованием несущей среды, называемой эфиром. Майкельсон объединился с ученым Эдвардом Морли в 1887 году, чтобы разработать экспериментальный метод обнаружения эфира путем наблюдения за относительными изменениями скорости света, когда Земля завершает свой оборот вокруг Солнца. Для достижения этой цели они разработали интерферометр, который разделяет луч света и перенаправляет отдельные лучи по двум различным путям, каждая длиной более 10 метров, используя сложную матрицу зеркал. Майкельсон и Морли пришли к выводу, что если Земля движется через эфирную среду, то луч, отражающийся взад и вперед перпендикулярно потоку эфира, должен будет проходить дальше, чем луч, отражающийся параллельно эфиру. Результатом была бы задержка в одном из световых лучей, которую можно было бы обнаружить, когда лучи рекомбинировались посредством интерференции.

      Экспериментальный аппарат, построенный Майкельсоном и Морли, был массивным (см. рис. 5). Установленный на медленно вращающейся каменной плите площадью более пяти квадратных футов и толщиной 14 дюймов, прибор был дополнительно защищен расположенным ниже слоем ртути, который действовал как амортизатор без трения, устраняя вибрации Земли. После того, как плита была приведена в движение, достигнув максимальной скорости 10 оборотов в час, потребовались часы, чтобы снова остановиться. Свет, проходящий через светоделитель и отраженный системой зеркал, исследовали с помощью микроскопа на наличие интерференционных полос, но ни одного из них не наблюдалось. Однако Майкельсон использовал свой интерферометр, чтобы точно определить скорость света на уровне 186 320 миль в секунду (299 853 километра в секунду), значение, которое стало стандартом на следующие 25 лет. Неспособность обнаружить изменение скорости света в эксперименте Майкельсона-Морли положила начало прекращению полемики об эфире, которая была окончательно положена в основу теорий Альберта Эйнштейна в начале двадцатого века.

      Специальная теория относительности Эйнштейна и скорость света

      В 1905 году Эйнштейн опубликовал свою Специальную теорию относительности, за которой в 1919 году последовала Общая теория относительности.15. Первая теория относилась к движению объектов с постоянной скоростью относительно друг друга, а вторая концентрировалась на ускорении и его связи с гравитацией. Поскольку они бросили вызов многим давним гипотезам, таким как закон движения Исаака Ньютона, теории Эйнштейна стали революционной силой в физике. Идея относительности воплощает в себе концепцию, что скорость объекта может быть определена только относительно положения наблюдателя. Например, человек, идущий внутри авиалайнера, движется со скоростью около одной мили в час в системе отсчета самолета (который сам движется со скоростью 600 миль в час). Однако наблюдателю с земли кажется, что человек движется со скоростью 601 миля в час.

      В своих расчетах Эйнштейн предположил, что скорость света, движущегося между двумя системами отсчета, остается одинаковой для наблюдателей в обеих точках. Поскольку наблюдатель в одном кадре использует свет для определения положения и скорости объектов в другом кадре, это меняет способ, которым наблюдатель может связать положение и скорость объектов. Эйнштейн использовал эту концепцию, чтобы вывести несколько важных формул, описывающих, как выглядят объекты в одной системе отсчета, если смотреть на них из другой, которая движется равномерно относительно первой. Его результаты привели к некоторым необычным выводам, хотя эффекты становятся заметными только тогда, когда относительная скорость объекта приближается к скорости света. Таким образом, основные следствия фундаментальных теорий Эйнштейна и его уравнения относительности, на которое часто ссылаются:

      E = mc 2

      можно резюмировать следующим образом:

      • Длина объекта уменьшается относительно наблюдателя по мере увеличения скорости этого объекта.

      • Когда система отсчета движется, временные интервалы становятся короче. Другими словами, космический путешественник, движущийся со скоростью света или близкой к ней, может покинуть Землю на много лет и вернуться, испытав промежуток времени всего в несколько месяцев.

      • Масса движущегося объекта увеличивается с увеличением его скорости, и когда скорость приближается к скорости света, масса приближается к бесконечности. По этой причине широко распространено мнение, что двигаться быстрее скорости света невозможно, поскольку для ускорения бесконечной массы потребуется бесконечное количество энергии.

      Хотя теория Эйнштейна повлияла на весь мир физики, она имела особенно важное значение для тех ученых, которые изучали свет. Теория объяснила, почему эксперимент Майкельсона-Морли не дал ожидаемых результатов, препятствуя дальнейшим серьезным научным исследованиям природы эфира как несущей среды. Он также продемонстрировал, что ничто не может двигаться быстрее скорости света в вакууме, и что эта скорость является постоянной и неизменной величиной. Тем временем ученые-экспериментаторы продолжали применять все более сложные инструменты, чтобы определить правильное значение скорости света и уменьшить погрешность ее измерения.

      Измерения скорости света

      Таблица 1
      0644 299 792,4588
      1667 ​​ Galileo Galilei ​​ Covered Lanterns ​​ 333.5 ​​
      1676 ​​ Ole Roemer ​​ Jupiter’s Moons ​​ 220,000 ​​
      1726 James Bradley ​​ Stellar Aberration ​​ 301,000 ​​
      1834 ​​ Charles Wheatstone ​​ Rotating Mirror ​​ 402,336 ​​
      1838 ​​ François Араго Вращающееся зеркало
      1849 Арман Физо ​​ Rotating Wheel ​​ 315,000 ​​
      1862 ​​ Leon Foucault ​​ Rotating Mirror ​​ 298,000 ​​
      1868 ​​ James Clerk Maxwell Теоретические расчеты. 0647 ​​ Rotating Mirror ​​ 299,990 ​​
      1879 ​​ Albert Michelson ​​ Rotating Mirror ​​ 299,910 ​​
      1888 ​​ Heinrich Rudolf Hertz Электромагнитное излучение0647 ​​ Electrical Measurements ​​ 300,000 ​​
      1890s ​​ Henry Rowland ​​ Spectroscopy ​​ 301,800 ​​
      1907 ​​ Edward Bennett Rosa and Noah Дорси Электрические измерения 299 788
      1923 Andre Mercier ​​ Electrical Measurements ​​ 299,795 ​​
      1926 ​​ Albert Michelson ​​ Rotating Mirror (Interferometer) ​​ 299,798 ​​
      1928 Август Каролус и Отто Миттельштадт Затвор Керра 299 778
      1932 to 1935 ​​ Michelson and Pease ​​ Rotating Mirror (Interferometer) ​​ 299,774 ​​
      1947 ​​ Louis Essen ​​ Cavity Resonator ​​ 299 792
      1949 Аслаксон К. И.9,792.4 ​​
      1951 ​​ Keith Davy Froome ​​ Radio Interferometer ​​ 299,792.75 ​​
      1973 ​​ Kenneth M. Evenson ​​ Laser ​​ 299 792,457
      1978 Питер Вудс и коллеги Лазер

      В конце девятнадцатого века достижения в области радио- и микроволновых технологий предоставили новые подходы к измерению скорости света. В 1888 году, более чем через 200 лет после первых астрономических наблюдений Ремера, немецкий физик Генрих Рудольф Герц измерил скорость радиоволн. Герц достиг значения около 300 000 километров в секунду, подтверждая теорию Джеймса Клерка Максвелла о том, что радиоволны и свет являются формами электромагнитного излучения. Дополнительные доказательства были собраны в ходе 1940-е и 1950-е годы, когда британские физики Кейт Дэви Фрум и Луи Эссен использовали соответственно радио и микроволны для более точного измерения скорости электромагнитного излучения.

      Максвеллу также приписывают определение скорости света и других форм электромагнитного излучения не посредством измерений, а путем математического вывода. Во время своих исследовательских попыток найти связь между электричеством и магнетизмом Максвелл предположил, что изменяющееся электрическое поле создает магнитное поле, что является обратным следствием закона Фарадея. Он предположил, что электромагнитные волны состоят из комбинированных колеблющихся электрических и магнитных волн, и рассчитал скорость этих волн в пространстве как:0036

      Скорость (V) = 1/(ε • µ) 1/2

      где ε — диэлектрическая проницаемость, а µ — проницаемость свободного пространства, две константы, которые можно измерить с относительно высокой степенью точности. . В результате получается значение, близкое к измеренной скорости света.

      В 1891 году, продолжая свои исследования скорости света и астрономии, Майкельсон создал крупномасштабный интерферометр с помощью телескопа-рефрактора в Ликской обсерватории в Калифорнии. Его наблюдения были основаны на задержке времени прихода света при просмотре удаленных объектов, таких как звезды, которые можно количественно проанализировать для измерения как размера небесных тел, так и скорости света. Почти 30 лет спустя Майкельсон перенес свои эксперименты в обсерваторию Маунт-Вилсон и применил те же методы к 100-дюймовому телескопу, крупнейшему в то время в мире.

      Включив восьмиугольное вращающееся зеркало в свой экспериментальный проект, Майкельсон получил значение скорости света 299 845 километров в секунду. Хотя Майкельсон умер до завершения своих экспериментов, его коллега по Маунт-Вилсон Фрэнсис Г. Пиз продолжал использовать инновационную технику для проведения исследований до 1930-х годов. С помощью модифицированного интерферометра Пиз провел многочисленные измерения в течение нескольких лет и, наконец, определил, что правильное значение скорости света равно 29.9 774 километра в секунду, самое близкое измерение, достигнутое на тот момент. Несколько лет спустя, в 1941 году, научное сообщество установило стандарт скорости света. Это значение, 299 773 километра в секунду, было основано на компиляции самых точных измерений того периода. На рис. 6 представлено графическое представление измерений скорости света за последние 200 лет.

      К концу 1960-х годов лазеры стали стабильными исследовательскими инструментами с точно определенными частотами и длинами волн. Вскоре стало очевидно, что одновременное измерение частоты и длины волны даст очень точное значение скорости света, подобное экспериментальному подходу, осуществленному Китом Дэви Фрумом с использованием микроволн в 1919 году.58. Несколько исследовательских групп в Соединенных Штатах и ​​других странах измерили частоту линии 633 нм гелий-неонового лазера, стабилизированного йодом, и получили очень точные результаты. В 1972 году Национальный институт стандартов и технологий использовал лазерную технологию для измерения скорости на уровне 299 792 458 метров в секунду (186 282 миль в секунду), что в конечном итоге привело к переопределению метра посредством высокоточной оценки скорости света.

      Начиная с 1676 прорывных попыток Ремера, скорость света измерялась по крайней мере 163 раза с использованием широкого спектра различных методов более чем 100 исследователями (см. Таблицу 1 для подборки методов, исследователей и дат). По мере совершенствования научных методов и устройств пределы погрешности оценок сужались, хотя скорость света существенно не изменилась со времени расчетов Ремера в семнадцатом веке. В 1983 году, более чем через 300 лет после первой серьезной попытки измерения, скорость света была определена как 299 792,458 километра в секунду Семнадцатым Всеобщим конгрессом мер и весов. Таким образом, метр определяется как расстояние, которое проходит свет за интервал времени 1/299 792 458 секунд. Однако в целом (даже во многих научных расчетах) скорость света округляется до 300 000 километров (или 186 000 миль) в секунду. Достижение стандартного значения скорости света было важно для создания международной системы единиц, которая позволила бы ученым со всего мира сравнивать свои данные и расчеты.

      Существуют небольшие разногласия по поводу того, существуют ли доказательства того, что скорость света замедлялась со времен Большого взрыва, когда он мог двигаться значительно быстрее, как предполагали некоторые исследователи. Хотя аргументы, представленные и опровергнутые, увековечивают этот спор, большинство ученых по-прежнему утверждают, что скорость света является постоянной величиной. Физики отмечают, что фактическая скорость света, измеренная Ремером и его последователями, существенно не изменилась, а скорее указывают на ряд усовершенствований в научных приборах, связанных с повышением точности измерений, используемых для определения скорости света. Сегодня расстояние между Юпитером и Землей известно с высокой степенью точности, как и диаметр Солнечной системы и орбитальные траектории планет. Когда исследователи применяют эти данные для переработки расчетов, сделанных за последние несколько столетий, они получают значения скорости света, сравнимые с теми, которые получены с помощью более современного и сложного оборудования.

      Соавторы

      Кеннет Р. Спринг — Научный консультант, Лусби, Мэриленд, 20657.

      Томас Дж. Феллерс , Лоуренс Д. Цукерман и Майкл У. Дэвидсон — 18 0 Национальная лаборатория сильного магнитного поля востока, 180 Пол Дирак Доктор, Университет штата Флорида, Таллахасси, Флорида, 32310.

      НАБОР ВОПРОСОВ 13A-ANS

      НАБОР ВОПРОСОВ 13A-ANS

      1 . Большая часть электромагнитного спектра состоит из видимый свет.

      А. Верно
      Б. Ложно

      2 . Электромагнитные волны состоят из

      А. сжатия и разрежения электромагнитных импульсов.
      В . колебания электрического и магнитного полей.
      C. частицы световой энергии.
      D. высокочастотные гравитационные волны.

      3 . Источником всех электромагнитных волн является

      .

      А. изменения уровней атомной энергии.
      B. колеблющиеся атомы.
      C. ускоряющие электрические заряды.
      D. кристаллические колебания.
      Е. все это.

      Обоснование: Ускоренный заряд производит электромагнитные волны.

      4 . Электромагнитные волны

      А . может путешествовать в вакууме.
      B. нужна среда для путешествия.

      Рассуждение: В отличие от звуковых волн, Электромагнитные волны не являются механическими волнами, поэтому им не нужна среда для распространения.

      5 . Основное различие между радиоволной и световой волной заключается в ее

      А. скорость.
      Б. длина волны.
      С. частота.
      д., все это.
      E. два таких.

      Обоснование: Длина волны и частота

      6 . Скорость света в стекле по сравнению со скоростью света в вакууме есть,

      А. На 100 % быстрее.
      Б. В 100 раз медленнее.
      C. На 50 % быстрее.
      D. На 33 % медленнее.

      7 . Какая из этих электромагнитных волн имеет наименьшую длину волны?

      А. радиоволны
      B. инфракрасные волны
      C. рентгеновские лучи
      D. ультрафиолетовые волны
      E. световые волны

      8 . По сравнению с радиоволнами скорость волн видимого света в вакуум

      А. меньше.
      Б. больше.
      С. то же.

      9 . Если электрон колеблется вверх и вниз 1000 раз в секунду, он генерирует электромагнитная волна, имеющая

      А. период 1000 с.
      Б. скорость 1000 м/с.
      C. Длина волны 1000 м.
      D. частота 1000 Гц.
      E. длина волны 1000 км.

      10 . Что из перечисленного принципиально отличается от остальных?

      А. звуковые волны
      Б. Рентген
      C. гамма-лучи
      D. световые волны
      E. радиоволны

      Рассуждение: Звуковые волны механичны волны. Остальные ответы представляют собой электромагнитные волны, которые НЕ являются механическими волнами.

      11 . Собственная частота атомов в стекле составляет

      А. Радиоволновая часть спектра.
      B. видимая часть спектра.
      С. ультрафиолетовая и инфракрасная части спектра.

      Обоснование: См. стр. 501 (последняя параграф обсуждает этот вопрос).

      12 . Когда ультрафиолетовый свет падает на стекло, атомы в стекле

      А. вынуждены вибрировать.
      В . резонировать.
      C. пропускать световую энергию практически в неизменном виде.
      D. свободно поглощают и переизлучают большую часть ультрафиолетового света.

      Обоснование: См. стр. 500.

      13 . Когда видимый свет падает на прозрачное стекло, атомы в стекле

      A. вынуждены вибрировать.
      Б. резонировать.
      C. преобразовать световую энергию во внутреннюю энергию.

      14 . Рассмотрим световую энергию, которая мгновенно поглощается стеклом и затем повторно излучается. По сравнению с поглощенным светом частота переизлучаемого свет

      А. значительно меньше.
      Б. несколько меньше.
      С. то же.
      Д. чуть больше.
      E. значительно больше

      Обоснование: См. стр. 501 (1-я параграф имеет некоторые обсуждения по этому поводу. )

      15 . По сравнению со средней скоростью в воздухе средняя скорость луча света в стекле

      А. подробнее.
      Б. меньше.
      С. то же.

      16 . Инфракрасные волны часто называют тепловыми волнами, потому что они

      А. исходят из относительно горячих источников.
      Б. состоят из частот более низких, чем у видимого света.
      C. вызывают резонанс в молекулах и увеличивают внутреннюю энергию в веществе.
      D. являются преобладающими волнами, излучаемыми солнцем

      17 . Солнечные ожоги производятся по

      А . ультрафиолетовый свет.
      Б. видимый свет.
      C. инфракрасный свет.
      д., все это.
      е. ни один из них.

      18 . Атмосфера Земли прозрачна для большинства волн

      А. вся инфракрасная часть спектра.
      В . видимая часть спектра.
      C. всей ультрафиолетовой части спектра.
      D. весь электромагнитный спектр.

      19 . Ощущение цвета возникает, когда свет падает на 9 глаз.0036

      А. стержни.
      B. конуса.
      С. оба.
      Д. ни.

      20 . Колбочек в сетчатке глаза

      .

      A. наиболее плотно упакованы в центре поля зрения.
      Б. равномерно распределены по полю зрения.
      C. сосредоточены по периферии зрения.

      21 . Раскаленные докрасна и раскаленные до синего цвета звезды кажутся глазу белыми, потому что

      A. Глаз плохо различает цвета ночью.
      B. они слишком тусклые, чтобы стрелять конусами.
      C. Эй слишком тусклые, чтобы стрелять стержнями.
      D. они подавлены чернотой ночного неба.
      E. Пики восприимчивости глаз находятся в желто-зеленой части спектра.

      22 . На периферии нашего зрения нас

      А. более чувствителен к низким частотам, чем к высоким.
      B. нечувствителен к цвету и движению.
      С . чувствительны к движению, но не различают цвета.
      D. чувствителен как к движению, так и к цвету.
      е. ни один из них.

      23 . Цвет видит

      А. Палочки в глазу.
      B. колбочки в глазу.
      C. оба из них.
      Д. Ни то, ни другое.

      24 . Информационные нервы соединяются с сетчаткой в ​​

      А. ямка.
      В . слепое пятно.
      C. роговица.
      Д. радужка.
      Е. периферия.

      25 . Вещи, видимые при лунном свете, обычно не окрашиваются из-за лунного света

      В

      А. не очень много цветов.
      B. слишком тусклый, чтобы активировать колбочки сетчатки.
      C. У фотонов недостаточно энергии, чтобы активировать сетчатку. конусы.
      д., все это.
      е. ни один из них.


      Калькулятор скорости волны в вакууме

      ✖Скорость волны представляет собой скорость волны, используемой для передачи через любую среду. FirstCosmic Velocity — SecondCosmic Velocity — ThirdEarth’s VelocityФут в часФут в минутуФут в секундуКилометр в часКилометр в минутуКилометр в секундуУзелУзел (Великобритания)Махмах (стандарт SI)Метр в часМетр в минутуМетр в секундуМиль в часМиль в минутуМиль в секундуМиллиметр в часМиллиметр в минутуМиллиметр в секундуМорская миля в деньNautical Mile Per Day Миля в часСкорость звука в чистой водеСкорость звука в морской воде (20°C и глубина 10 метров)Ярд в часЯрд в минутуЯрд в секунду

      +10%

      -10%

      ✖Показатель преломления среды определяется тем, как свет проходит через эту среду. Это безразмерная мера. Он определяет, насколько луч света может преломиться при переходе из одной среды в другую.

      ✖Скорость в вакууме — это скорость волны, когда волна проходит через вакуум.ⓘ Скорость волны в вакууме [V 0 ]

      Сантиметр в часСантиметр в минутуСантиметр в секундуКосмическая скорость — перваяКосмическая скорость — втораяКосмическая скорость — третья в часМиллиметр в минутуМиллиметр в секундуМорская миля в деньМорская миля в часСкорость звука в чистой водеСкорость звука в морской воде (20°C и глубина 10 метров)Ярд в часЯрд в минутуЯрд в секунду

      ⎘ Копировать

      👎

      Формула

      Перезагрузить

      👍

      Скорость волны в вакуумном растворе

      ШАГ 0: Сводка предварительного расчета

      ШАГ 1: Преобразование входных данных в базовые единицы

      Скорость волны: 50 м/с —> 50 м/с Преобразование не требуется
      Показатель преломления: 1,333 —> нет Требуется преобразование

      ШАГ 2: Вычисление формулы

      ШАГ 3: Преобразование результата в единицу измерения

      66,65 Метр в секунду —> Преобразование не требуется

      < 10+ Калькуляторы поправок EDM 9-6*Парциальное давление водяного пара))*((273,15+Температура)/(0,269578*(Групповой показатель преломления для стандартных условий-1))) Идти

      Парциальное давление водяного пара с учетом показателя преломления

      Парциальное давление водяного пара = (((0,269578*(Групповой показатель преломления для стандартных условий-1)*Барометрическое давление)/(273,15+Температура))-Групповой показатель преломления+1)*((273,15+Температура)/(11,27* 10^-6)) Идти 9-6 Идти

      Разница температур при заданном парциальном давлении

      Изменение температуры = (давление насыщенного пара воды – парциальное давление водяного пара)/0,7 Идти

      Парциальное давление водяного пара с учетом влияния температуры

      Парциальное давление водяного пара = Давление насыщенного пара воды-0,7*Изменение температуры Идти

      Скорость волны в среде

      Скорость волны = скорость в вакууме/показатель преломления Идти

      Скорость волны в вакууме

      Скорость в вакууме = скорость волны * показатель преломления Идти

      Формула скорости волны в вакууме

      Скорость в вакууме = скорость волны * показатель преломления
      V 0 = V*RI

      Что такое показатель преломления?

      В оптике показатель преломления (также известный как показатель преломления или показатель преломления) материала представляет собой безразмерное число, описывающее скорость прохождения света через материал. Показатель преломления определяет, насколько искривляется или преломляется путь света при попадании в материал.

      Как рассчитать скорость волны в вакууме?

      Калькулятор скорости волны в вакууме использует Скорость в вакууме = Скорость волны * Показатель преломления для расчета скорости в вакууме. Скорость волны в вакууме определяется как скорость волны, которая распространяется в вакууме. Вакуум – это пространство, лишенное материи. Слово происходит от латинского прилагательного «vacuus», означающего «пустой» или «пустой». Скорость в вакууме обозначается символом V 0 .

      Как рассчитать скорость волны в вакууме с помощью этого онлайн-калькулятора? Чтобы использовать этот онлайн-калькулятор для скорости волны в вакууме, введите скорость волны (V) и показатель преломления (RI) и нажмите кнопку расчета. Вот как можно объяснить расчет скорости волны в вакууме с заданными входными значениями -> 66,65 = 50 * 1,333 .

      Часто задаваемые вопросы

      Что такое скорость волны в вакууме?

      Скорость волны в вакууме определяется как скорость волны, распространяющейся в вакууме. Вакуум – это пространство, лишенное материи. Слово происходит от латинского прилагательного «vacuus» для «пустого» или «пустого» и представлено как 9.0644 V 0 = V*RI или Скорость в вакууме = Скорость волны*Показатель преломления . Скорость волны представляет собой скорость волны, используемой для передачи через любую среду. Показатель преломления среды определяется тем, как свет проходит через эту среду. Это безразмерная мера. Он определяет, насколько световой луч может преломиться при переходе из одной среды в другую.

      Как рассчитать скорость волны в вакууме?

      Скорость волны в вакууме определяется как скорость волны, распространяющейся в вакууме. Вакуум – это пространство, лишенное материи. Слово происходит от латинского прилагательного «vacuus» для «пустого» или «пустого», рассчитанного с использованием Скорость в вакууме = Скорость волны*Показатель преломления . Для расчета скорости волны в вакууме вам понадобится скорость волны (V) и показатель преломления (RI) . С помощью нашего инструмента вам нужно ввести соответствующее значение скорости волны и показателя преломления и нажать кнопку расчета. Вы также можете выбрать единицы измерения (если есть) для ввода (ов) и вывода.

      Поделиться

      Скопировано!

      Волна Аспект Света: Интерференция | Физика |

      Цели обучения

      К концу этого раздела вы сможете:

      • Обсуждать волновой характер света.
      • Определите изменения, когда свет входит в среду.

      Мы знаем, что видимый свет — это электромагнитная волна, на которую реагируют наши глаза. Как и все другие электромагнитные волны, она подчиняется уравнению c = , где c = 3 × 10 8 м/с — скорость света в вакууме, f — частота электромагнитных волн, а λ — их длина волны. Диапазон видимых длин волн составляет приблизительно от 380 до 760 нм. Как и для всех волн, свет распространяется прямолинейно и действует как луч, когда взаимодействует с объектами, в несколько раз превышающими его длину волны. Однако, когда он взаимодействует с более мелкими объектами, он заметно проявляет свои волновые характеристики. Интерференция является отличительной чертой волны, и на рисунке 1 можно увидеть как лучевые, так и волновые характеристики света. Лазерный луч, испускаемый обсерваторией, представляет собой луч, движущийся по прямой линии. Однако прохождение луча чистой длины волны через вертикальные щели с размером, близким к длине волны луча, обнаруживает волновой характер света, поскольку луч распространяется по горизонтали в виде узора из ярких и темных областей, вызванных систематической конструктивной и деструктивной интерференцией. Вместо того, чтобы распространяться, луч будет продолжать двигаться прямо после прохождения через щели.

      Создание соединений: волны

      Наиболее верным признаком волны является интерференция. Эта волновая характеристика наиболее заметна, когда волна взаимодействует с объектом, который невелик по сравнению с длиной волны. Интерференция наблюдается для водных волн, звуковых волн, световых волн и (как мы увидим в специальной теории относительности) для волн материи, таких как электроны, рассеянные кристаллом.

      Рис. 1. (a) Лазерный луч, испускаемый обсерваторией, действует как луч, движущийся по прямой линии. Этот лазерный луч исходит от обсерватории Параналь Европейской южной обсерватории. (кредит: Юрий Белецкий, Европейская южная обсерватория) (б) Лазерный луч, проходящий через сетку вертикальных щелей, создает интерференционную картину, характерную для волны. (кредит: Шимон и Слава Рыбка, Wikimedia Commons)

      Свет имеет волновые характеристики в различных средах, а также в вакууме. Когда свет переходит из вакуума в какую-либо среду, например воду, его скорость и длина волны меняются, но его частота f остается прежней. (Мы можем думать о свете как о вынужденных колебаниях, которые должны иметь частоту исходного источника. ) Скорость света в среде равна

      v=cnv=\frac{c}{n}\\v=nc​

      , где n — его показатель преломления. Если мы разделим обе части уравнения c  на n , мы получаем

      cn=v=fλn\frac{c}{n}=v=\frac{f\lambda}{n}\\nc​=v=nfλ

      . Отсюда следует, что v n , где λ n — длина волны в среде и что

      λn=λn\lambda_\fractext{n}}= lambda}{n}\\λn​=nλ​

      , где λ — длина волны в вакууме, а n — показатель преломления среды. Следовательно, длина волны света в любой среде меньше, чем в вакууме. В воде, например, у которой = 1,333, диапазон видимых длин волн

      380 нм1,333\frac{380\text{ нм}}{1,333}\\1,333380 нм​

       до

      760 нм1,333 text{нм}}{1,333}\\1,333760 нм​

      или  λ n  = от 285 до 570 нм. Хотя длины волн меняются при перемещении из одной среды в другую, цвета не меняются, поскольку цвета связаны с частотой.

      Резюме раздела

      Концептуальные вопросы

      1. Какие экспериментальные данные указывают на то, что свет является волной?
      2. Приведите пример волновой характеристики света, которую легко наблюдать вне лаборатории.

      Задачи и упражнения

      1. Покажите, что при переходе света из воздуха в воду его длина волны уменьшается в 0,750 раза по сравнению с исходным значением.
      2. Найдите диапазон видимых длин волн света в краун-стекле.
      3. Каков показатель преломления материала, для которого длина волны света в 0,671 раза больше его значения в вакууме? Определите вероятное вещество.
      4. Анализ эффекта интерференции в прозрачном твердом теле показывает, что длина волны света в твердом теле составляет 329 нм. Зная, что этот свет исходит от гелий-неонового лазера и имеет длину волны 633 нм в воздухе, является ли это вещество цирконом или алмазом?
      5. Каково отношение толщин краун-стекла и воды, которые содержали бы одинаковое количество длин волн света?

      Глоссарий

      Длина волны в среде:  

      λ_n=λn{\lambda}\text{\textunderscore}{n}=\text{}\lambda{}{n}\\λ_n=λn

      , где λ — длина волны в вакууме, а n — показатель преломления среды

      Избранные решения задач и упражнений

      1.

      alexxlab

      Добавить комментарий

      Ваш адрес email не будет опубликован. Обязательные поля помечены *